Anda di halaman 1dari 125

Internal Medicine- Dermatology

A 66 year old man has the toe nails shown in figure below. His nails have been like this for at least 25 years. He is troubled by recurring cracking
between the toes and redness and scaling on the soles of both feet. He would like to have this treated. You believe this is a fungal infection of the nail
and will take a sample.
The most likely organism that would cause this infection is which of the following:

1)
2)
3)
4)
5)

Trichophyton rubrum
Epidermophyton floccosum
Malassezia furfur
Microsporum gypseum
Candida albicans

The correct answer is choice A

Tricophyton rubrum is the causative fungal pathogen in the majority of onychomycosis cases. This condition is difficult to eradicate, is poorly responsive
to topical interventions, but fairly responsive to systemic therapy with the broad spectrum antifungals terbenafine (Lamisil) and fluconazole
(Diflucan). One topical prescription medication has been approved for the treatment of fungal nail infections. The medication is Penlac Nail Lacquer
(ciclopirox topical 8% solution). Overall, Penlac has a low rate of effectiveness. However, it can be a treatment option in patients who are at high risk
for complications with the use of oral medication therapy. Oral medication therapy has the highest rate of effectiveness, ranging from 50% to over
80%. However, treatment failure and reoccurrences are not uncommon.

Oral medication tends to have the highest success rate in the treatment of fingernails and with younger patients. Lamisil (terbinafine) is generally
considered first line treatment when using an oral medication. It has a higher rate of efficacy when compared to other oral medications and few serious
side effects as well as fewer medication interactions. Patient with infected fingernails take 250 mg daily for six weeks and those with infected toenails
take 250 mg daily for twelve weeks. There are other oral medications approved for the treatment of onychomycosis. These include Sporanox
(itraconazole) and Diflucan (Fluconazole). Routine blood tests are recommended for patients being treated with any of these oral medications, as they
can cause liver problems.

Most persons with onychomycosis suffer from co-infection with tinea pedisa condition which almost always results from co-infection with Trichophyton
rubrum, Trichophyton mentagrophytes, and Epidermophyton floccosum.

There are four patterns of nail infections. More than one pattern may occur simultaneously.

1. Distal subungual onychomycosis is the most commonly seen pattern. The distal (free edge at the top of the nail) gets infected with fungi and as a
result, turns white to yellow in color. This discoloration eventually spreads to the entire nail. The infection also causes the nail to separate from
the nailbed.

2. Proximal subungal onychomycosis is the most commonly seen pattern in immunocompromised patients. The fungi invade the proximal-cuticle

area and infect the nail from below. The surface of the nail remains intact. However, the infection causes the nail to separate from the nailbed.

3. White superficial onychomycosis is caused by surface infection of the nail plate. This often starts as dull, white spots on the nail surface and
eventually, the entire nail becomes soft, dry and powdery. However, the nail plate does not separate from the nailbed.

4. Candidal onychomycosis is a nail-plate infection. It is seen primarily in patients with the rare disease; chronic mucocutaneous candidiasis. The
nail plate turns yellowish-brown in color and thickens. This condition usually involves all the fingernails.

Epidermophyton floccosum (choice b) is incorrect because, while this agent is responsible for the condition most commonly associated with
onychomycosistinea pedis (see answer a discussion)it is not a causative pathogen in onychomycosis.

Malessezia furfur (choice c) is incorrect because it is the pathogen that causes tinea versicolor. This condition is a benign, recurring superficial
cutaneous fungal infection usually characterized by hypopigmented or hyperpigmented macules and patches on the chest and the back.

Microsporum gypseum (choice d) is incorrect because it is a causative or contributing fungal pathogen in tinea capitis (ringworm), seborrheic
dermatosis, and other scalp dermatoses but does not cause nail pathology.

Candida albicans (choice e) is incorrect because while this opportunistic yeast causes and contributes to numerous mild and severe medical conditions
including diaper dermatitis, vaginal candidiasis, and candidal sepsis, amongst others, it plays no role in the development of onychomycosis.

A 7 year old girl is brought to your office by her mother because of a rash that appeared 3 days ago. Her temperature is 37.2 degrees C (99 degrees F)
and her face has an intense erythematous eruption involving her bilateral cheeks. Development history is normal and immunization is up to date. She is
on n medications and has a history that is only significant for recurrent otitis media.

The most likely etiologic agent is which of the following:

1)
2)
3)
4)
5)

Adenovirus
Rotavirus
Parvovirus
Coxsackievirus
Echovirus

The correct answer is Choice C.

Parvovirus B19 infection is common in school-aged and younger children who attend daycare facilities. In general, children transmit the virus to parents
and siblings. In young children, the antibody seroprevalence ranges from 5-10%. This increases to 50% in adolescents and approaches 90% in the
elderly. Common symptoms of parvovirus B19 (B19V) infection include a mild nonspecific prodromal illness that may consist of fever (15-30% of
patients), malaise, headache, myalgia, nausea, and rhinorrhea; typically beginning 5-7 days after initial infection. These symptoms correspond to the
initial viremia and dissipate in 2-3 days. Approximately 1 week later, a bright red macular exanthem appears on the cheeks (slapped cheek) and is
often associated with circumoral pallor. A diffuse maculopapular rash (erythematous macules and papules that become confluent, giving a lacy or
reticulated appearance) can appear 1-4 days later and fades to a lacy erythematous rash, which may be pruritic and may spread gradually toward the
distal extremities. Most seropositive patients have no history of this classic biphasic illness. The clinical symptoms widely vary, and the classic "slapped
cheek" rash is much more common in young children. Often, Parvovirus B19 can cause self-limited arthralgias, lasting 3 weeks; but may persist for
several months or years.

Adenovirus (Choice A) most commonly causes illness of the respiratory system; however, depending on the infecting serotype, they may also cause
various other illnesses, such as gastroenteritis, conjunctivitis, cystitis, and rash illness. Symptoms of respiratory illness caused by adenovirus infection
range from the common cold syndrome to pneumonia, croup, and bronchitis. Patients with compromised immune systems are especially susceptible to
severe complications of adenovirus infection. Despite Adenovirus can cause exanthem it does not begin with slapped cheek appearance that excludes
the Adenovirus (Choice A).

Rotavirus (Choice B) is one of several viruses known to cause gastroenteritis.Symptoms usually begin within 2 days of exposure and include anorexia,
low-grade fever, watery, bloodless diarrhea, vomiting, abdominal cramps. The symptoms. mentioned above are not consistent with the case.

Coxsackieviruses (Choice D) can cause several ilnesses.Thus, group A coxsackieviruses tend to infect the skin and mucous membranes, causing
herpangina, acute hemorrhagic conjunctivitis (AHC), and hand-foot-and-mouth (HFM) disease. Group B coxsackieviruses tend to infect the heart, pleura,
pancreas, and liver, causing pleurodynia, myocarditis, pericarditis, and hepatitis. Both group A and group B coxsackieviruses can cause nonspecific
febrile illnesses, rashes, upper respiratory tract disease, and aseptic meningitis. Hand-foot-and-mouth disease (HFMD) is a systemic infection caused by
coxsackievirus A16, characterized by ulcerative oral lesions and a vesicular exanthem on the distal extremities in association with mild constitutional
symptoms. Herpangina is caused by coxsackievirus A16, 8, 10, 22; also coxsackie group B (strains 14). echoviruses, and other enteroviruses. It
usually affects children <5 years and is prevalent in late summer and early fall in temperate climates. The symptoms are a sudden onset of fever,
malaise, headache, anorexia, dysphagia, and sore throat. The case does not describe he symtom typical for Coxsackieviruses type A nor type B.

Echoviruses (EV) (Choice E) are common human pathogens that cause a range of illnesses, from minor febrile illness to severe, potentially fatal
conditions (eg, aseptic meningitis, encephalitis, paralysis, myocarditis). Individual serotypes have different temporal patterns of circulation and cause
different clinical manifestations. EV causes a remarkable range of diseases. Benign forms of disease such as nonspecific exanthems and herpangina do
not warrant major diagnostic or therapeutic actions. Severe forms of disease, such as meningitis, encephalitis, neonatal sepsis, myocarditis, and chronic
infection with meningoencephalitis in patients with altered immunity, are strong reasons for concern. Skin rashes are more common with EV infections.
Exanthems may be maculopapular, morbilliform, macular, petechial, or papulopustular in nature. Skin findings with EV infections are self-limited and
without sequelae. The clinical picture of the EV infection does not include the slapped cheek appearance that allows to omit this choice.

You are a senior resident in pediatrics and get called by the nurse to evaluate the rash of a newborn in the nursery who is 24 hours old. On examination
there is a widespread rash, however the child is doing well and vitals are within normal limits. The rash is on the trunk, back, face, and proximal
extremities without mucosal, palmar and plantar involvement. It is represented by erythematous, irregular, blanchable and variable in size macules with
a few wheales, papules, and pustules. You order a smear and Wrights stain of the intralesional material, which shows greater than 90% eosinophils and

variable numbers of neutrophils.


This rash and associated findings are most consistent with which of the following diagnosis?

1)
2)
3)
4)
5)

Infantile Acropustulosis
Erythema toxicum neonatorum
Transient Neonatal Pustular Melanosis
Cutis Marmorata Telangiectatica Congenita
Neonatal Lupus Erythematosus

The correct answer is Choice B.

Erythema toxicum neonatorum (ETN) (also known as "Erythema toxicum, and "Toxic erythema of the newborn") is a common rash in neonates. It
appears in up to half of newborns carried to term and usually occurs in the first 24 to 48 hours of life. The clinical picture is represented by the Fleabitten rash of few to hundreds of erythematous, irregular, blanchable and variable in size macules, wheales, papules, and pustules. On histology there
is folliculitis with eosinophils (few neutrophils, no spongiosis). Erythema toxicum neonatorum always spares palms and soles and usually resolves within
1 to 2 weeks. The cause of erythema toxicum neonatorum is unknown.

Infantale acropustolosis (Choice A) is not correct because it is represented by crops of pruritic vesiculopustules on the hands and feet of infants and
occurs in crops every 2 to 4 weeks. The etiology of the Infantale acropustolosis may be associated with a reaction to previous scabies exposure,
however scabies prep is negative.

The Transient Neonatal Pustular Melanosis (TNPM) (Choice C) is also incorrect answer and on presentation consists of very superficial vesicles and

sterile pustules.TNPM is seen at birth in up to 4 % healthy, term newborns. The elements typically rupture with desquamation leaves charcteristic
residual hyperpigmented macules generally within first 2 days of life.On histology there is intracorneal and subcorneal neutrophilic spongiosis.

Cutis Marmorata Telangiectatica Congenita (Choice D) is a wrong answer too. Cutis Marmorata Telangiectatica Congenita a congenital condition that
characterized reticulate purple network. Most cases occur sporadically and often associated with limb girth discrepancy.

Neonatal Lupus Erythematosus (NLE) (Choice E) is caused by the influence of mothers antibodies (anti-Ro, anti-La or U1-ribonucleoprotein) on the
fetus. Mothers are usually asymptomatic at the time of delivery. The rash typically consists of well-demarcated, annular, erythematous, scalling plaques.
Congenital heart block can be at 15% to 30%. The clinical picture is not consistent with the description, so it can be excluded.

A 29 year old woman comes to you to check a mole on her back. She is worried that her tanning bed use prior to a trip to Cancun has given her
melanoma. On exam, the mole was 2 cm in diameter. While you recommend removal, the patient would rather not worry about the problem, so she
does not return to you. One year later, she is brought in by her room-mate because the mole has now ulcerated.
Which of the following statements are true:
1)
2)
3)
4)
5)

This is a good prognostic sign as it indicates infection rather than malignancy


Besides a wide resection she will likely need a regional lymph node dissection
Adjuvant chemotherapy would increase her chance at cure
There is little likelihood of wide-spread metastasis
This melanoma is likely at Clarks level I in depth

The correct answer is choice C

The clinical scenario above is consistent with the diagnosis of malignant melanoma. The clinical indicators that suggest the likelihood of a pigmented
lesion being a malignant melanoma are remembered by the pneumonic ABCDE. A is for asymmetry: melanomas tend to be asymmetrical in shape; B
is for Border: melanomas usually have irregular borders; C is for Color: malignant melanoma lesions often demonstrate a variegation of colors; D is
for Diameter: melanomas usually exceed 6 mm in diameter; the letter E can be added to this pneumonic because most melanomas, if watched over
time, will demonstrate evolutionary change: in other words, their appearance alters and they tend to become more ominous-appearing with time, with
changes including necrosis and ulceration.

If we assume that this patient had a 2.0 cm melanoma 1 year ago, it is quite likely that over the past year it has likely undergone gone evolutionary
changes including enlargement and what appears to be necrosis.

This is a good prognostic sign as it indicates infection rather than malignancy (choice a) is incorrect because this patients lesion has ulcerateda
degenerative process that would be categorized as worrisome evolutionary change. This suggests that the original lesion was a melanoma.

Besides a wide resection she will likely need a regional lymph node dissection (choice b) is not correct because this has not been show to improve
survival in melanoma. She is likely at Clark Level III in her melanoma staging.

Treatment for this stage of melanoma is wide excision with adjuvant chemotherapy. There is little likelihood of wide-spread metastasis (choice d) is not
correct because if her lesion is greater than 2.0 cm, then she has a 57% chance of regional metastases and a 15% likelihood of distal metastases. The
likelihood increases if her lesion diameter exceeds 4.0 cm.

Figure 1: Clark's and Breslow levels

Figure 2: Survival data for Melanoma based on Clarks and Breslow thickness

A 72-year-old man in good health presents to his family physician with fever and malaise that have lasted for several days. He also reports that he has
developed an itchy, burning rash on the right side of his chest and back. All vital signs are within normal limits. Physical examination reveals an
erythematous maculopapular rash with vesicles on the right side of the patients back and chest wall. Based on history and physical examination, the
patient is diagnosed with herpes zoster infection.
If the examination had revealed evidence of herpes zoster ophthalmicus, which of the following cranial nerves would be involved?
1) Facial nerve
2) Oculomotor nerve
3) Trigeminal nerve

4) Trochlear nerve
5) Petrosal nerve
The correct answer is Choice C

The trigeminal nerve supplies the eye by means of its ophthalmic division. The oculomotor nerve supplies the musculature responsible for eye
movement.

Although herpes zoster rarely involves more than one dermatome, or perhaps a bit more frequently adjacent dermatomes, in immunocompromised
patients such as those undergoing chemotherapy or perhaps afflicted with HIV or AIDS, extension to more dangerous distant sites becomes more
plausable. Sites such as the eyes or ears could present severe consequences. Herpes zoster ophthalmicus, for example is a serious condition with a
threat of visual loss. With this in mind, the physician should further evaluate the patient for visual and otologic complications related to herpes zoster
infection such as eye redness, facial lesions, or external auditory canal lesions.

Following initial ophthalmic infection, the varicella-zoster virus (VZV) enters the dorsal root ganglia of the trigeminal nerve, or in the otic form the
geniculate nerve, where it remains latent for the lifetime of the individual.

In immunocompetent patients, specific antibodies (immunoglobulins G, M, and A) appear more rapidly and reach higher titers during reactivation
(herpes zoster) than during the primary infection.

The appearance of the rash coincides with a profound VZV-specific T-cell proliferation. Interferon-alpha production appears when the herpes zoster
resolves. The end result is a long-lasting, enhanced, cell-mediated immunity response to VZV.

You are asked to examine a 2 day old newborn baby girl in the nursery. She was the product of a normal, spontaneous vaginal delivery without any
known trauma or use of instrumentation. Upon examination of the skin, multiple findings are noted. The rest of her physical exam is within normal
limits.

Which one of the following dermatological findings is most concerning?


1)
2)
3)
4)
5)

densely packed, tiny, white pustules on the nose


scattered small areas of erythema with a central white, raised dot
a reddish area over the left eye region
a large, bluish discoloration on the sacrum
a reddish area to the central, lower occipital region

The correct answer is choice C.

The dermatological exam of a neonate can be plentiful with findings, most of which are benign. It is important to be able to recognize the benign
findings along with what needs to be monitored and/or worked up further.

The finding that is most concerning is a reddish area over the left eye region. With this description, one must worry about a nevus flammeus
(commonly called a port-wine stain because of its color). A port-wine stain is usually pink to reddish in color, flat, and varies in size and distribution
though they are often found on the face in a unilateral position. Port-wine stains do not fade as the child ages as do most benign lesions and can be
indicative of an underlying syndrome or pathology, such as Sturge-Weber or Klippel-Trenaunay-Weber syndromes. When they involve the eye
region, as in this vignette, there is a concern for glaucoma in the affected individual. Most port-wine stains can be effectively treated with laser therapy.

Milia (choice A) appear as tiny whitish or pearly bumps commonly distributed around the nose and cheeks. It is caused by the trapping of dead cells at
the surface of the skin. Milia are benign in nature and disappear within a few weeks of life.

Erythema toxicum (choice B), despite its name, is a benign rash that usually appears within the first few days of life. It is characterized by scattered
areas of erythema surrounding a central, white papule and is often found on the face, arms, and trunk. The fluid in the central papules, if examined,
can be seen to contain eosinophils. Erythema toxicum usually disappears on its own without treatment by a few weeks of life.

Mongolian spots (choice D) are a benign finding in a newborn caused by the migration of melanocytes under the skin. These spots can appear
anywhere but are most typical in the area of the sacrum. Mongolian spots are most prevalent in darker-skinned individuals. It is important to note that
Mongolian spots are not bruises, and if any diagnostic confusion is present, care should be taken to obtain a good history in order to differentiate the

two entities.

A nevus simplex (often referred to as a stork bite, angel kiss, or salmon patch; choice E) is sometimes difficult to differentiate from a nevus flammeus
(port-wine stain). A nevus simplex is a benign, reddish discoloration of the skin that is temporary and generally disappears as the infant grows older, as
opposed to a port-wine stain, which grows with the child. It is common on the forehead, base of the occiput/neck, and eyelids. They are often midline
findings, whereas a port-wine stain is more often unilateral.

A 55-year-old woman presents to her physician with complaints of a moderately painful rash that she first noticed the previous day. On examination, a
band of grouped erythematous vesicles is noted over the right flank in the T12 dermatome.
Reasonable treatment options at this time (either alone or in combination with other agents) include all of the following EXCEPT:
1)
2)
3)
4)
5)

Prednisone
Acyclovir
Carbamazepine
Opiate analgesics
Valacyclovir

The correct answer is choice C

The patient in our example is suffering from varicella zoster, commonly known as shingles. Caused by recurrence of varicella virus (a member of the
herpes virus family) as a result of decline in cell-mediated immunity with age, zoster causes a painful unilateral vesicular rash that follows a single
dermatome and occurs more frequently in the aged, immunosuppressed, and those suffering from multiple medical concerns. Approximately 20 percent
of the general population will experience shingles during their lifetime.

Carbamazepine (Tegretol) (choice C) has no role in the management of acute zoster but, rather, receives the majority of its use in the management of
epilepsy, with best results seen in patients suffering from partial seizures with complex symptomatology (psychomotor, temporal lobe). It is also utilized
in patients suffering from generalized tonic-clonic seizures (grand mal) and mixed pattern epilepsy. Petit Mal (absence) seizures do not respond well to
Tegretol. Tegretol is also used in prophylaxis for numerous chronic pain syndromes, including trigeminal neuralgia and migraine headaches, and
postherpetic neuralgia, a potential complication of varicella zoster (see discussion below).

Prednisone (Choice A) is correct because this powerful anti-inflammatory drug is used in the management of acute zoster when inflammation is
contributing to severe pain concerns. Caution must be employed by the physician to balance the benefits of utilizing the drug against Prednisones
immunosuppressive qualities, as immunosuppression is a risk factor for zoster development.

Acyclovir (Choice B), brand name Zovirax, is correct because this product is indicated for the management of acute herpes zoster. Its use reduces the
severity and symptom length of acute episodes, is used in recurrence prophylaxis and ,if employed early in the course of the acute illness can help
prevent postherpetic neuralgiaa markedly distressing chronic pain syndrome along the affected dermatome that can develop in up to 1/3 of zoster
patients.

Acute zoster can be a very painful condition and narcotic analgesia (Choice C) is therefore an correct answer because it, along with anti-inflammatory
medications, is often required for pain control.

Valacyclovir (Choice D), brand name Valtrex, is correct because it, along with famcyclovir (Famvir), belongs to the same group of anti-viral agents as
Zovirax and is a mainstay of acute zoster treatment.

Figure 1: Acute treatment of HSV review

A previously healthy 7-year-old girl has been brought to the emergency with high fever, rashes and bilateral conjunctival injection. On examination the
temperature is 40C (104 F), respiratory rate is 24/min, heart rate is 130/min and blood pressure is 99/62 mm Hg. The girl is irritable and her lips are
dry. Her chest is clear to auscultation. Examnation of the oral cavity reveals a shiny red tongue with prominent papillae ("strawberry tongue").
"Strawberry tongue" is associated with which of the following conditions:

1)
2)
3)
4)
5)

Scarlet fever
Kawasaki syndrome
Parvovirus B19 infection
A and B
All of the Above (A, B, and C)

The correct answer is Choice D

Kawasaki disease (Choice B) (mucocutaneous lymph node syndrome) is a multisystemic acute large- and medium-vessel vasculitis that primarily affects
young children (80% are < 5 years of age) with predilection for those of Asian ancestry. This condition is divided into acute,subacute, and chronic
phases.

Diagnostic criteria:
Lasts 12 weeks and presents with the following symptoms:

1. Fever persiting (usually > 40C (104 F) for at least five days AND 4 of the criteria below:

Bilateral, nonexudative, painless conjunctivitis.

Polymorphous rash (primarily truncal).

Cervical lymphadenopathy (often unilateral, with at least one node >1.5 cm).

Diffuse mucous membrane erythema (e.g., strawberry tongue, red fissured lips).

Erythema of the palms and soles; indurative edema of the hands and feet; late desquamation of the fingertips (in subacute phase).

Scarlet fever (Choice A) is an infection caused by toxin-producing group A beta hemolytic streptococci (GABHS) found in secretions and discharge from
the nose, ears, throat, and skin. The mucous membranes usually are bright red, and scattered petechiae and small red papular lesions on the soft
palate are often present. During the first days of infection, the tongue is heavily coated with a white membrane through which edematous red papillae
protrude (classic appearance of white strawberry tongue). By day 4 or 5, the white membrane sloughs off, revealing a shiny red tongue with prominent
papillae (strawberry tongue). Red, edematous, exudative tonsils, as shown below, are typically observed if the infection originates in this area. The
exudative pharyngitis typical of scarlet fever. Although the tongue is somewhat out of focus, the whitish coating observed early in scarlet fever is visible.

The characteristic exanthem consists of a fine erythematous punctate eruption that appears within 1-4 days following the onset of the illness. It first
appears on the upper trunk and axillae and then becomes generalized, although it is usually more prominent in flexural areas, such as the axillae,
popliteal fossae, and inguinal folds. It may also appear more intense at dependent sites and sites of pressure, such as the buttocks.The face is usually

flushed, and circumoral pallor is observed.

The cutaneous rash, shown below, lasts for 4-5 days, followed by fine desquamation, the extent and duration of which is directly related to the severity
of the eruption. Desquamation of the palms is a frequently observed self-limited manifestation of scarlet fever present in the healing period following
resolution of the infection and acute eruption.

Parvovirus B19 infection (Choice C) is common in school-aged and younger children who attend daycare facilities. In general, children transmit the virus
to parents and siblings. In young children, the antibody seroprevalence ranges from 5-10%. This increases to 50% in adolescents and approaches 90%
in the elderly. Common symptoms of parvovirus B19 (B19V) infection include a mild nonspecific prodromal illness that may consist of fever (15-30% of
patients), malaise, headache, myalgia, nausea, and rhinorrhea; typically beginning 5-7 days after initial infection.

These symptoms correspond to the initial viremia and dissipate in 2-3 days. Approximately 1 week later, a bright red macular exanthem appears on the
cheeks (slapped cheek) and is often associated with circumoral pallor. A diffuse maculopapular rash (erythematous macules and papules that become
confluent, giving a lacy or reticulated appearance) can appear 1-4 days later and fades to a lacy erythematous rash, which may be pruritic and may
spread gradually toward the distal extremities. Most seropositive patients have no history of this classic biphasic illness. The clinical symptoms widely
vary, and the classic "slapped cheek" rash is much more common in young children. Often, Parvovirus B19 can cause self-limited arthralgias, lasting 3
weeks; but may persist for several months or years. The "strawberry tongue" is not a feature of the Parvovirus B19 infection.

A 35 year old pregnant woman is referred to your outpatient clinic for management of systemic lupus eythematosus. You decide that her chronic
management regimen will include prednisone, so you explain the possible side effect risks of chronic corticosteroid therapy to her.
All of the following are true regarding corticosteroids except:
1)
2)
3)
4)
5)

Hydrocortisone has the least glucocorticoid potency


Dexamethasone has the highest mineralcorticoid potency
Methylprednisolone has a higher potency then prednisone
Prednisolone is 5 times more potent than cortisone
Pregnancy category C

The correct answer is choice B.

The corticosteroids are an extremely important class of drugs utilized in the management of many acute and chronic disease processes. However,
potential side effect concerns limit their usage and require that providers be diligent requiring their prescribing. The most frequently prescribed product
in the class is prednisone because of its low cost resulting from its generic status. Systemic steroids include prednisone, prednisolone,
methylprednisolone, beclamethasone, betamethasone, dexamethasone, fludrocortisone, hydrocortisone and triamcinolone.

If systemic steroids have been prescribed for one month or less, side effects are rarely serious. However the following problems may arise:

Sleep disturbance

Increased appetite

Weight gain

Psychological effects, including increased or decreased energy

Rare but more worrisome side effects of a short course of corticosteroids include: mania, psychosis, heart failure, peptic ulceration, diabetes and
aseptic necrosis of the hip.

Nearly everyone on systemic steroids for more than a month suffers from some adverse effects. These may include any of the following problems,
which are not listed in any particular order of importance.

Reduction of cortisol production. During and after steroid treatment, the adrenal gland produces less of its own cortisol, resulting from
hypopituitary-pituitary-adrenal (HPA) axis suppression. For up to twelve months after the steroids are stopped, the lack of steroid response to
stress such as infection or trauma could result in severe illness.

Osteoporosis (thinning of the bones) particularly in smokers, postmenopausal women, the elderly, those who are underweight or immobile, and
patients with diabetes or lung problems. Osteoporosis may result in fractures of the spine, ribs or hip joint with minimal trauma. These occur
after the first year in 10-20% of patients treated with more than 7.5mg prednisone daily. It is estimated that up to 50% of patients on long term
oral corticosteroids will develop bone fractures.

Reduction in growth in children, which may not catch up when the steroids are discontinued (but it usually does).

Muscle weakness, especially of the shoulder muscles and thighs.

Rarely, avascular necrosis of the femoral head (destruction of the hip joint).

Precipitation or aggravation of diabetes mellitus (high blood sugar).

Increase in circulating blood fat (triglycerides).

Redistribution of body fat: moon face, buffalo hump and truncal obesity.

Salt retention: leg swelling, raised blood pressure, weight increase and heart failure.

Shakiness and tremor.

Eye disease, particularly glaucoma (increased intraocular pressure) and posterior subcapsular cataracts.

Psychological effects including insomnia, mood changes, increased energy, excitement, delirium or depression.

Headaches and raised intracranial pressure.

Increased susceptibility to internal infections, especially when high doses are prescribed (e.g. tuberculosis).

Peptic ulceration, especially common in those also taking anti-inflammatory medications.

There are also side effects from reducing the dose; these include tiredness, headaches, muscle and joint aches and depression.

From Figure 2, the potency, mineralcorticoid activity and half-life of corticosteroids is summarized to provide the explanation for the correct and
incorrect choices of the question.

Figure 1: Basic structure of corticosteroids

A 58 year-old female patient presents to a dermatology clinic complaining of erythematous itchy patches over her malar eminences that had developed
over the past 3 weeks. She also has recently developed an unexplained rash in the exposed parts of her body which seem to worsen when she goes out
during the daytime. She also complains of pain, tenderness and swelling in her bilateral fingers. Urine analysis was completed and revealed proteinuria
>0.5g/l (+++). Blood tests revealed anemia, leukopenia, lymphopenia and thrombocytopenia. In addition, Anti-nuclear antibody (ANA) was elevated at
1:160 and she was positive for anti-double stranded DNA antibodies.
After reviewing her medical history, you were convinced this was the result of an adverse drug reaction. A particular drug was discontinued and this was
followed by rapid improvement of both clinical and laboratory findings.
Which of the following drugs has the highest association with this condition:
1)
2)
3)
4)
5)
6)

Furosemide
Procainamide
Metformin
Minocycline
Enalapril
D-Penicillamine

The correct answer is choice B

DILE is caused by long-term use of certain medications (from months to years). It is reversible it resolves after discontinuation of the drug. Some
drugs can cause DILE, while others may cause a flare of pre-existing SLE. More than 80 drugs have been associated with DILE. Most cases have been
associated with these three: Procainamide, hydralazine, quinidine

Risk Factors

Slow drug acetylation phenotype

Genetic deficiency of the enzyme N-acetyltransferase

Results in slower deactivation of many drugs (including procainamide, hydralazine)

Elderly patients are more likely to develop DILE: Due to decreased metabolism and increased medication usage

No evidence that individuals with SLE are more likely to develop DILE

As with SLE, most people with drug-induced lupus develop antinuclear antibodies, or ANAs, although those with a form of drug-induced lupus related to
quinidine often are ANA-negative. The ANAs in drug-induced lupus are primarily autoantibodies that are able to react with a histone-DNA complex,
which is the major component of the nucleus of all cells.

A special laboratory test to detect certain antibodies to this histone-DNA complex is a sensitive marker for lupus-like disease brought on by many drugs.
Hydralazine is the exception, as only about one-third of people with DILE have this type of anti-histone antibody.

The most important aspect of treating drug-induced lupus is to recognize the medication that is likely to be causing the problems. Its use can then be
discontinued. This step is often sufficient to improve the symptoms within a few days, which will indicate that symptoms were drug-induced. Individuals
will probably improve more quickly if non-steroidal anti-inflammatory drugs (NSAIDs) or steroids are then used.

Table 1: Drugs associated with DILE

A 56 year old man had a small, slowly growing nodule on his chin during the past 3 years. The lesion is 1.3 cm in diameter, the center is ulcerated, and
the border is waxy.
Examination of tissue obtained on excision of the lesion is most likely to show which of the following?
1)
2)
3)
4)
5)

actinic keratosis
basal cell carcinoma
malignant melanoma
seborrheic keratosis
squamous cell carcinoma

The correct answer is Choice B

Basal cell carcinoma (basalioma, BCC) is the most common skin cancer and the most common human malignancy in general, with a continuously
increasing incidence. In most cases, BCC develops on chronically sun-exposed skin in elderly people, most commonly in the head and neck region.
Besides chronic UV radiation, other risk factors for the development of BCC include sun bed use, family history of skin cancer, skin type 1 and 2, a
tendency to freckle in childhood, immunosuppression, previous radiotherapy, and chronic exposure to certain toxic substances such as inorganic arsenic.
There are numerous variations in clinical presentation of BCC, such as nodular BCC, ulcerating BCC, pigmented BCC, sclerosing BCC, superficial BCC,
and fibroepithelioma of Pinkus. Each varies in terms of clinical presentation, histopathology and aggressive behavior. Treatment modalities for BCC
include surgical excision, cryosurgery, curettage, electrodessication, radiotherapy, photodynamic therapy, topical cytostatics, and immunomodulators. If
left untreated or inadequately treated, BCC may become invasive and locally destructive, although it very rarely metastasizes.

BCCs may resemble squamous cell carcinoma, keratoacanthoma, actinic keratosis, or molluscum contagiosum. It may be clinically impossible to
distinguish a flesh-colored intradermal nevus from a BCC. BCCs show varying pigmentation irrespective of their behavior. Pigmented BCC may mimic
pigmented seborrheic keratosis, nevi, angiokeratoma, superficial spreading or nodular malignant melanoma. Sclerosing BCC can easily be overlooked
due to its resemblance to a scar. Superficial BCC is often misdiagnosed as psoriasis or the nummular type of eczema, but lesions lack the silvery scale of
psoriasis and, on contrary to eczema, BCC has sharp border. An ulcer on the lower extremities may represent a BCC or squamous cell carcinoma.
According to some authors, it takes several years to recognize this skin tumor and make appropriate diagnosis in these patients. In addition,
fibroepithelioma of Pinkus may sometimes mimic amelanotic melanoma.

Different treatment modalities for BCC include surgical excision, cryosurgery, curettage and cautery, electrodessication, radiotherapy, photodynamic
therapy, topical cytostatics, and immunomodulators. The treatment of BCC is determined by the size and location of the tumor, clinical variant, patient

age and comorbidities such as diabetes mellitus and hypertension.

Actinic keratosis (Choice A) is not correct because these lesions, which also occur on sun-exposed skin of older persons and are slow growing, are
notable for their hard, horny surface and are unlikely to ulcerate.

Malignant melanoma (Choice B) is incorrect because these potentially deadly lesions are characterized by hyperpigmentation, are flat, but are not waxy
in their presentation. Their morphology is easiest remembered by the pneumonic ABCDE, which represents: A for asymmetry (irregular shape); B for
border (border irregular and interrupted); C for color (dark and variegated color/s); D for diameter (size larger than 0.5 cm suspicious for malignancy);
and E for evolutionary change (change in presentation over time such as color change, ulceration, spread).

Seborrheic keratosis (Choice D) is incorrect because these non-malignant lesions that tend to occur on the face, head, neck and shoulder area are very
unlikely to ulcerate. Major concerns with these lesions are cosmetic and irritation (get caught on jewelry), resulting in patient requests for removal.

Squamous cell carcinoma (Choice E) is incorrect because of the word waxy in the description of the lesion. Waxy is a buzzword associated with basal
cell carcinomas. Otherwise, squamous cell carcinomas of the skin share commonalities with basal cell carcinomas in that they tend to occur on sunexposed areas of the skin, tend to progress over time, and can ulcerate. However, unlike basal cell carcinomas, these lesions have malignant potential
and can cause death via dissemination, an occurrence that is fortunately fairly rare.

A 14-year-old boy had a febrile illness with severe pharyngitis treated presumptively with amoxicillin. Ten days later he continued to have fever and
pharyngitis and has developed an extensive morbilliform eruption over the trunk and extremities, including the palms and soles.
Which of the following studies would be most helpful in establishing the cause of his illness?
1)
2)
3)
4)
5)

Weil-Felix test
Blood cultures for meningococcus
ASO titer
RPR test
EBV monospot testing

The correct answer is Choice E

Infectious mononucleosis (IM) is caused by Epstein-Barr virus (EBV) and has been recognized as a clinical syndrome consisting of fever, pharyngitis,
and adenopathy. EBV infectious mononucleosis is a common cause of viral pharyngitis in patients of all ages, but it is particularly frequent in young
adults. In the United States, approximately 50% of the population seroconverts before age 5 years, with much of the rest seroconverting in adolescence
or young adulthood. Most patients with Epstein-Barr virus (EBV) infectious mononucleosis are asymptomatic and, therefore, have few if any symptoms.
Most adults (approximately 90%) show serological evidence of previous EBV infection.

The incubation period of EBV infectious mononucleosis is 1-2 months. Many patients cannot recall close contact with individuals with pharyngitis.
Virtually all patients with EBV infectious mononucleosis report fatigue and prolonged malaise. A sore throat is second only to fatigue and malaise as a
presenting symptom. Fever is usually present and is low grade, but chills are relatively uncommon. Arthralgias and myalgias occur but are less common
than in other viral infectious diseases. Nausea and anorexia, without vomiting, are common symptoms. The classic presentation of EBV infectious
mononucleosis in children and young adults consists of the triad of fever, pharyngitis, and lymphadenopathy. Early signs include fever,
lymphadenopathy, pharyngitis, rash, and/or periorbital edema. Relative bradycardia has been described in some patients with EBV mononucleosis, but it
is not a constant finding.

Later physical findings include hepatomegaly, palatal petechiae, jaundice, uvular edema, splenomegaly, and rarely (1-2%), findings associated with
splenic rupture. CNS findings associated with EBV mononucleosis are rare but usually occur later in the course of the illness. There is a linkage between
administration of amoxicillin and morbilliform rash at patients suffered from IM.

Epstein-Barr virus (EBV) infection induces specific antibodies to EBV and various unrelated non-EBV heterophile antibodies. These heterophile antibodies
react to antigens from animal RBCs. The heterophile antibodies are sensitive and specific for EBV heterophile antibodies, they are present at peak levels
2-6 weeks after primary EBV infection, and they may remain positive in low levels for up to a year. The latex agglutination assay, which is the basis of
the Monospot test using horse RBCs, is highly specific. Sensitivity is 85%, and specificity is 100%.

A Weil-Felix reaction (Choice A) is a type of agglutination test in which patients serum is tested for agglutinins to O antigen of certain non-motile
Proteus and rickettsial strains (OX19, OX2, OXk). OX19, OX2 are strains of Proteus vulgaris. The test is used in diagnostics certain ricketsial diseases.

Blood cultures for meningococcus (Choice B) is used for the diagnosis of meningococcal infection. So, it is a wrong answer.

Antistreptolysin O titre (ASOT) (Choice C) - titre of (serum) antistreptolysin O antibodies is a blood test used to assist in the diagnosis of a streptococcal
infection or indicate a past exposure to streptococci.

Rapid Plasma Reagin (RPR) (Choice D) refers to a type of test that looks for non-specific antibodies in the blood of the patient that may indicate that the
organism (Treponema pallidum) that causes syphilis is present. The term "reagin" means that this test does not look for antibodies against the actual
bacterium, but rather for antibodies against substances released by cells when they are damaged by T. pallidum.

A 25 year old woman has a dark brown to black dome-shaped papule on her shin. It is about 5 mm in diameter and non tender. It never bleeds and she
hasnt notices much change in size. It has probably been present for a few years. It is firm to touch and when squeezed, the lesion dimples in the
center.
Which of the following is the most likely diagnosis?

1)
2)
3)
4)
5)

seborrheic keratosis
dermatofibroma
malignant melanoma
actinic keratosis
junctional melanocytic nevus

The correct answer is Choice B

Dermatofibromas are common cutaneous nodules of unknown etiology that occur more frequently in women, are usually seen on the extremities
(predominantly the lower legs), and are usually asymptomatic. Some lesions, however, will cause pruritus and tenderness. Because of the common
nature of dermatofibromas, even though most are not painful there are still enough persons with painful lesions to make this the most common painful
skin lesion seen by office physicians. Finally, these lesions manifest a characteristic tethering of the overlying epidermis to the underlying lesion with
lateral compression, called the dimple sign.

Seborrheic keratosis (Choice A) is incorrect because these non-malignant lesions are usually seen in older persons and tend to occur on the face, head,
neck and shoulder area. Major concerns with these lesions are cosmetic and irritation (get caught on jewelry), resulting in patient requests for removal.
It would be very unlikely to see one on a young person's leg.

Malignant melanoma (Choice C) is incorrect because the lesion in this example demonstrates a dimple sign. In addition, dermatofibromas are fairly easy
to distinguish from malignant melanomas in that they tend to be raised. Melanomas, potentially deadly lesions, are hyperpigmented, macular (flat)
lesions. Their morphology is easiest remembered by the pneumonic ABCDE, which represents: A for asymmetry (irregular shape); B for border (border
irregular and interrupted); C for color (dark and variegated color/s); D for diameter (size larger than 0.5 cm suspicious for malignancy); and E for
evolutionary change (change in presentation over time such as color change, ulceration, spread).

Actinic keratosis (Choice D) is not correct because these lesions, like seborrheic keratosis, are seen almost exclusively in older persons and are notable
for their hard, horny surface.

Junctional melanocytic nevus (Choice E) is incorrect because these lesionsnevi composed of nevus cells located along the junction of the epithelium
and underlying dermiswhich are macular, brown to black in color, do not demonstrate gender predilection and do not demonstrate the dimple sign.

You are a senior resident in pediatrics who is examining the 6-month-old female. During the examination you note a macular blue-gray pigmentation on
the sacral area of the infant. Besides this pigmentation there are no other findings.The child is doing well and vitals are within normal limits.
Development history is normal and immunization is up to date. Also, parents told you that this pigmentation has been since the birth. You have
diagnosed a Mongolian spot.

Which of the following statements Mongolian spots in children are true:

1)
2)
3)
4)
5)

Seen in Down's syndrome


Normally disappears by 4 years
Most common site is the buttocks
Due to its risk of melanoma, requires surgical intervention with surgical margins
Blue tinge is explained by superficial depth of pigment

The correct answer is Choice B

Mongolian spot refers to a macular blue-gray pigmentation usually on the sacral area of healthy infants. Mongolian spot is usually present at birth or
appears within the first weeks of life. Mongolian spot typically disappears spontaneously within 4 years but can persist for life. Mongolian spot is a
congenital, developmental condition exclusively involving the skin. Mongolian spot results from entrapment of melanocytes in the dermis during their
migration from the neural crest into the epidermis. More than 90% of Native Americans, 80% of Asians, and 70% of Hispanics have Mongolian spots;

less than 10% of whites have Mongolian spots.

Mongolian spots consist of blue-gray macular pigmentation. The distinctive skin discoloration is due to the deep placement of the pigment in the dermis,
which imparts a bluish tone to the skin from the Tyndall effect* of scattered light. Typically, it is a few centimeters in diameter, although much larger
lesions also can occur. Lesions may be solitary or numerous.Commonly, Mongolian spot involves the lumbosacral area, but the buttocks (Choice C),
flanks, and shoulders may be affected in extensive lesions. Generalized Mongolian spots involving large areas covering the entire posterior or anterior
trunk and the extremities have been reported.

Several variants exist including the following:

Persistent Mongolian spots are larger, have sharper margins, and persist for many years.

Aberrant Mongolian spots involve unusual sites such as the face or extremities.

Persistent aberrant Mongolian spots also are referred to as macular-type blue nevi.

Superimposed Mongolian spots, in which a darker Mongolian spot overlies a lighter one, have been described.

Mongolian spots have been associated with cleft lip, spinal meningeal tumor, melanoma, phakomatosis pigmentovascularis and Sjgren-Larsson
syndrome (one case report). A few cases of extensive Mongolian spots have been reported with inborn errors of metabolism, the most common being
Hurler syndrome, followed by gangliosidosis type 1, Niemann-Pick disease, Hunter syndrome, and mannosidosis. In such cases, the Mongolian spots are
likely to persist rather than resolve. However, there are no associations with the Downs syndrome (Choice A).

Opaque cosmetics may be used as camouflage for Mongolian spots. Also, Mongolian spots do not require surgery as they have no inherent risk of
melanoma (Choice D).

*Tyndall effect (TE)


The change that light undergoes as it passes through a turbid mediumeg, skin, causing the colors of the spectrum to scatter; colors with a longer
wavelengthred, orange and yellow tend to continue traveling forward while those with a shorter wavelengthblue, indigo and violet scatter to the side
and backward; the TE explains why a subcutaneous lesion, which should have a red-brown hue due to hemorrhage or melanin deposition, has a blue
tinge that omits superficial depth of pigment (Choice E).

A 70-year-old woman presents to her family physician requesting more information about the herpes zoster vaccine. Her friends have told her about the
new vaccine, and she wants to know if she is a candidate to receive it. She had chicken pox as a child but has not had shingles. She notes that she is
concerned about shingles because of the debilitating pain associated with this condition.
The herpes zoster vaccine is made from which of the following components?
1)
2)
3)
4)
5)

Inactivated virus
Live attenuated virus
Polysaccharides
Toxoid
Conjugated antibody

The correct answer is choice B

ZOSTAVAX is a live attenuated virus vaccine indicated for prevention of herpes zoster (shingles) in individuals 60 years of age and older. It was
developed by Merck & Co. Local reactions at the injection's site were generally mild.

Zostovax was approved and licensed by the U.S. Food and Drug Administration (FDA) in May, 2006. The FDA recommended it only for adults 60 years
of age or older. It has been recently licensed in the United States and was subsequently licensed by the European Union for the prevention of herpes
zoster and herpes zoster-related postherpetic neuralgia (PHN) in individuals 50 years of age.

As demonstrated by the Shingles Prevention Study (SPS), Zostavax has been shown to reduce the incidence of herpes zoster and PHN in adults 60 years
of age and to lessen acute and chronic pain associated with the disease, presumably through boosting of VZV-specific immune responses. The SPS
demonstrated that the VZV antibody (Ab) response correlated with vaccine effect in preventing herpes zoster.

During a study of approximately 38,000 people throughout the United States who were 60 years of age and older about half received Zostavax and half
placebo. Participants were followed for about three years to see if they developed shingles and if they did, how long the pain lasted. Results found that
overall the vaccine reduced the occurrence of herpes zoster by about 50%. The effect was highest at 64% in people between the ages 60-69, but then
declined with increasing age; to 41% for the 70-79 age group, and 18% for those 80 years of age and older.

In those who were vaccinated with Zostavax, but still developed the disease, the duration of pain was a bit shorter than for those who received a
placebo. Specifically, the pain of those in the Zostavax group lasted on average about 20 days while for those who received placebo about 22 days. The
severity of the pain did not appear to differ among the two groups.

A 3 month old infant is presented with erythema and thick yellow-orange greasy scales and crust on the scalp and eyebrows. Eczematous lesions are
also present on the arms and trunk with sparing palms and soles.With scraping the scales easily separate from the lesions, however, there were not any
point hemorrhage after scales removal. KOH preparation was negative. The child is doing well and vitals are within normal limits. Development history is
normal and immunization is up to date. Also, parents deny any rashes and itching among people who contacted to the infant.
The most likely diagnosis includes which of the following:

1)
2)
3)
4)
5)

Scabies
Candidiasis
Contact dermatitis
Seborrheic dermatitis
Psoriasis

The correct answer is Choice D.

Seborrheic dermatitis (SD) is a very common chronic dermatosis characterized by redness and scaling and occurring in regions where the sebaceous
glands are most active, such as the face (central forehead, glabella, eyebrows, nasolabial folds) and scalp ("cradle cap"), the presternal area, and in the
body folds. The rash is represented by greasy, salmon-colored patches. Malassezia furfur is said to play a role in the pathogenesis, and the response to
topical ketoconazole and selenium sulfide is some indication that this yeast may be pathogenic.

(Choice A) Scabies is not correct answer because there is no evidence of infection (itching, rash) among childs caregivers and contacts. Also, palms and
soles are not spared in the case of infant/young children scabies.

Negative KOH preparation rules out Candidiasis (Choice B).

The clinical picture and history are not consistent with the contact dermatitis (Choice C) that is the responce of the skin to an irritant (acids, alkalis, etc.)

Negative test for the Psoriatic Triad omits (Choice E) psoriasis :

Psoriatic Triad.

Lesions of bright pink or a red color, covered with abundant silvery white scales. Lesions have pronounced edges. With scraping the scales easily
separate from the lesions;

Under the separated scales there appears a reddish moist tender semi-transparent skin;

With further scraping at the moist tender skin there will appear small droplets of blood (point hemorrhage)

A 10 year old male presents to the emergency room with a 1-day history of erosions of the lips, conjunctiva, along with a fever and a blistering rash. He
was previously healthy and recently was diagnosed with a sinus infection. When he presented to the emergency room, he was toxic looking and needed
to be intubated because of a GCS of 6. On examination, he was hemodynamically unstable with a blood pressure of 90/40, pulse was 150 and a
temperature over 40 degrees Celcius. His family informs you that he had just completed a 10 day course of cotrimoxazole antibiotics for the sinus
infection that he was diagnosed with. You are now suspecting Stevens-Johnson Syndrome (SJS).
Which of the
1)
2)
3)

following drug classes is most commonly associated with SJS:


Anticonvulsants
Barbiturates
Sulfonamides

4) Penicillins
5) Quinolones

The correct answer is choice C

Stevens Johnson syndrome must be seriously considered in this case. The extremely suggestive history and utilization of cotrimoxazole as therapy both
point to this immune-complexmediated hypersensitivity complex as the culprit.

A significant relationship of medications to the risk of Stevens-Johnson syndrome and toxic epidermal necrolysis has been identified in children less
than15 years of age.

Four previously highly suspected drug risk factors for Stevens-Johnson syndrome and toxic epidermal necrolysis in children have been confirmed.
Included are antiinfective sulfonamides (30% and most common cause), phenobarbital, carbamazepine, and lamotrigen (anticonvulsants are the second
most common cause). One important unexpected risk factor has also been suspected. It seems that acetaminophen (paracetamol) use might have the
potential for increasing the risk of either Stevens-Johnson syndrome or toxic epidermal necrolysis.

Only skin biopsy can definitively establish the diagnosis of Stevens-Johnson syndrome. CBC may reveal a normal white blood cell count or a nonspecific
leukocytosis. If WBC is severely elevated, the possibility of a superimposed bacterial infection exists. In case serious bacterial infection and
sepsis accompanies the disease morbidity and mortality become real threats. Skin and blood cultures are therefore advised.

Evaluation of renal function and examination of the urine for blood should also be checked. Other laboratory tests should include electrolytes and other
chemistries to help manage related problems. Cultures of blood, urine, and wounds are indicated when an infection is clinically suspected.

Table 1: Drugs and their risk of causing SJS

An obese 61 year-old female presents with a typical venous stasis ulcer that is 2.5 cm in diameter located superomedial to the medial malleolus.
Physical examination of her legs reveals bilateral stigmata of chronic venous disease: brawny edema, hyperpigmentation, scattered venous varicosities,
and scarring from previously healed venous stasis concerns.
Which of the following treatment modalities is most advisable?

1)
2)
3)
4)
5)

varicose vein ligation and stripping


broad spectrum antibiotics
compression therapy
anticoagulation (eg. Coumadin)
trental (pentoxifylline) for vasoactive effects

The correct answer is Choice C

The sequela to CVI that results in the greatest morbidity, mortality, and patient debilitation is chronic venous leg ulceration. Venous ulceration accounts
for 70 to 80% of lower limb ulceration; ischemic arterial ulcers, rheumatologic disorders, local trophic effects, unrecognized cancer, and other more
exotic causes constitute the balance. A venous ulcer is defined as an area of discontinuity of the epidermis, persisting for 4 weeks or more, as a result of
venous hypertension. Typically, these lesions occur around and above the medial malleolus and below the bulge of the gastrocnemius muscles, where
venous pressure is maximal due to the underlying presence of large perforating veins. While CVI is capable of producing leg ulcers by itself, there may
be other underlying comorbidities that contribution to the breakdown of skin. In patients presenting with venous leg ulcers, about 14% of have a
significant arterial component.

Chronic venous hypertension leads to venous stasis ulcers. White cell activation, protein leakage from pressurized capillaries, and cytokine imbalances
have all been implicated as indirect effects of venous hypertension that contribute to dermal changes seen in chronic venous insufficiency. Pressure
elevations (like venous hypertension) can directly result in altered cell function and morphology that may contribute to the delayed wound healing seen
in patients with venous ulcers.

The cause of skin breakdown that leads to stasis ulcers in patients who suffer from venous stasis concerns is inadequate extremity circulation caused by
excessive fluid-related pressure at the areas of concern . The series of occurrences that result in venous varicosity are as follows:

(1) excessive pressure due to overweight/other causes on distal vasculature results in collapse of venous valves, thus decreasing the distal
venous vasculatures ability to remove blood/fluid from the extremity; this results in;

(2) pooling/fluid retention in the extremities which, as it increases, causes;

(3) pressure throughout the extremity which;

(4) impedes circulation to the limb and its cutaneous compartments, thus;

(5) decreasing the ability of the skin to heal from minor abrasions and making it vulnerable to general breakdown and related concerns such as
ulcers and infection.

The role of compression stockings in venous stasis concerns is that compression by the stockings helps compensate for the lost squeezing power of
the veins that no longer exists due to the breakdown and loss of the veins valvular mechanisms designed to help return fluid from the distal limb to the
central circulation. In essence, the stockings do the squeezing that the venous valves no longer can.

Venous disease should be confirmed by clinical assessment and on special investigations. If this is the case then compression treatment should be
commenced. It should only be applied after the arteries have been assessed by measuring the ankle-brachial index. This is because if compression is
applied and the arteries are badly diseased, this can damage the ulcer and the leg, and make matters worse. It would also be very painful.

Varicose vein stripping and ligation (Choice A) is incorrect because these procedures are performed for the purposes of pain reduction and cosmesis.
Surgical procedures in patients with comprised vasculature are inadvisable due to increased risk for post-operative infectiona concern that can result
in marked morbidities including amputation and death in such patients.

Antibiotics (Choice B) is incorrect due to the fact that the primary etiology is less likely infectious although they can still play important roles in
treatment. Overtreatment with antibiotics is likely to lead to problems with antibiotic resistance developing in bacteria that are present, making the
future treatment of infection even more difficult.

Anticoagulation (Choice D) is incorrect because the underlying cause of venous stasis or venous stasis ulcers has nothing to do with hypercoagulation;
the cause is mechanical due to pooling of fluid.

Pentoxifylline (Trental) (Choice E) a drug that targets inflammatory cytokine release, leukocyte activation, and platelet aggregation at the microcirculatory level, is occasionally used for chronic venous insufficiency, but the reported efficacy is variable, and the benefit, when present, is generally
small. In a meta-analysis of five trials involving a total of 445 patients, the combination of compression and pentoxifylline (1200 mg per day in most
studies) modestly improved ulcer healing as compared with compression and placebo (relative risk, 1.3; 95% confidence interval, 1.1 to 1.5). The most
common side effect of this medication is mild gastrointestinal discomfort.

Table 1: Review of Causes of Ulceration

You are a senior resident in pediatrics who is examining the 2-month-old female. During the examination you note a 2-cm bright red nodular plaque on
her right thigh. Besides this nodular plaque there are no other findings.The child is doing well and vitals are within normal limits. Development history is
normal and immunization is up to date.
When counseling the parents regarding treatment of this lesion, which of the following is true?

1)
2)
3)
4)
5)

Surgery is the treatment of choice


Topical steroids are the most effective therapy
50% of these lesions will spontaneously resolve by age 9 years
95% of these lesions will spontaneously resolve by age 9 years
The lesion will most likely be present until the child is in their 20s

The correct answer is Choice D

Hemangioma of Infancy (HOI) (also known as strawberry, cherry, capillary hemangioma) is the most common tumor of infancy. Soft, bright red to deep
purple, compressible. On diascopy, does not blanch completely and on exam, clinically presents as a nodule or plaque, 1 to 8 cm in size.
The incidence in newborns is between 1 and 2.5%; in white children by 1 year of age it is 10%. Females more affected than males by a 3 to 1
ratio. HOI is a localized proliferative process of angioblastic mesenchyme. The initial proliferative phase lasts from 3 to 9 months, sometimes

more. HOI usually enlarge rapidly during the first year. In a subsequent phase of involution about the 95% of HOI regress, and this occurs gradually
over 2 to 6 years and is usually complete by the age of 10. Involution varies greatly between individuals and is not correlated with size, location, or
appearance of the lesion.

Surgery (Choice A) is an incorrect wrong answer because it is not a treatment of choice. Surgery is indicated in HOI that obstruct vital structures (eyes,
ears, larynx) and those (<1%) that are life threatening. Other options include topical intralesional steroids (Choice B), pulsed dye laser and
cryosurgery. However, treatment modalities mentioned above can have serious side effects such as scarring (laser, surgery, cryodestruction) and skin
atrophy (intralesional steroids). The rate of spontanious resolution is greater than 50% (Choice C) and it is usually complete earlier than their
20s (Choice E).

New research has now shown that Propanolol, if used within the first year of life, can have some benefit effects in the natural course of HOI. Dosing
includes 2 mg kg day given in divided doses every 12 hours. Blood pressure, pulse and blood sugar are mandatory tests that should be done in office
when starting this medication.

A 4-year-old child is febrile for 5 days and has bilateral conjunctival injection, cervical lymphadenopathy, dry, cracked, cherry red lips, swelling of the
hands and feet, and truncal polymorphous rash. On examination the child has a temperature of 40 C (104 F), heart rate of 125/min, respirarory rate
of 26/min and a blood pressure of 90/60 mm Hg. The child is quite irritable but is alert.
The most likely diagnosis is which of the following:

1)
2)
3)
4)
5)

Kawasaki disease
Rubella
Rubeola
staphylococcal scalded skin syndrome
toxic shock syndrome

The correct answer is Choice A

Kawasaki disease (Choice A) (mucocutaneous lymph node syndrome) is a multisystemic acute large- and medium-vessel vasculitis that primarily affects
young children (80% are < 5 years of age) with predilection for those of Asian ancestry. This condition is divided into acute,subacute, and chronic
phases.
Diagnostic criteria include the following:

Lasts 12 weeks and presents with the following symptoms:

Fever persiting (usually > 40C (104 F) for at least five days AND 4 of the criteria below:

Bilateral, nonexudative, painless conjunctivitis.

Polymorphous rash (primarily truncal).

Cervical lymphadenopathy (often unilateral, with at least one node >1.5 cm).

Diffuse mucous membrane erythema (e.g., strawberry tongue, red fissured lips).

Erythema of the palms and soles; indurative edema of the hands and feet; late desquamation of the fingertips (in subacute phase).

Rubella (Choice B) is usually a mild viral (RNA virus classified as a Rubivirus in the Togaviridae family) illness involving the skin, the lymph nodes, and,
less commonly, the joints.In children, a prodrome may not be present. The rash may be the first manifestation. In adults, fever, sore throat, and rhinitis
may be present. The exanthem begins as discrete macules on the face that spread to the neck, the trunk, and the extremities. The macules may
coalesce on the trunk. Appearance of the rash corresponds with the appearance of the rubella-specific antibody. The exanthem lasts 1-3 days, first
leaving the face, and may be followed by desquamation. On occasion, a nonspecific enanthem (Forchheimer spots) of pinpoint red macules and
petechiae can be seen over the soft palate and the uvula just before or with the exanthem.

Measles (Rubeolla) (Choice C) is one of the most contagious infectious diseases, with at least a 90% secondary infection rate in susceptible domestic
contacts, and can affect people of all ages, despite being considered primarily a childhood illness. It is caused by RNA virus of the genus Morbillivirus
within the family Paramyxoviridae. The prodrome is characterized by a high fever (often >104o F [40o C]) for 3-5 days and the classic triad comprises
nonproductive cough, coryza, and conjunctivitis. Other associated prodromal symptoms may include photophobia, periorbital edema, myalgias, and
malaise. Typical for measles is an enanthem (Koplik spots). Koplik spots are white spots on the buccal cheeks just opposite the first and second upper
molars and on the labial mucosa that occur 2-4 days after the onset of the prodrome and precede the exanthem by 24-48 hours. Koplik spots are
described as grains of sand on an erythematous base and are pathognomonic for measles, although their absence does not exclude the diagnosis.
Exanthem is represented by blanching, erythematous macules and papules begin on the face at the hairline, on the sides of the neck, and behind the
ears. Within 48 hours, they coalesce into patches and plaques that spread cephalocaudally to the trunk and extremities, including the palms and soles,
while beginning to regress cephalocaudally, starting from the head and neck. The eruption may also be petechial or ecchymotic in nature. The
exanthem last 5-7 days before fading into coppery brown hyperpigmented patches, which then desquamate.

Staphylococcal scalded skin syndrome (SSSS) (Choice D) is a toxin-mediated type of exfoliative dermatitis caused by toxigenic strains of Staphylococcus
aureus.Staphylococcal scalded skin syndrome (SSSS) originates from a focus of infection that may be a purulent conjunctivitis, otitis media, or occult
nasopharyngeal infection and begins with fever, irritability, and a generalized, faint, orange-red, macular erythema with cutaneous tenderness.Within
24-48 hours, the rash progresses from a scarlatiniform to a blistering eruption. Characteristic tissue paperlike wrinkling of the epidermis is followed by
the appearance of large, flaccid bullae in the axillae, in the groin, and around the body orifices.Subsequent generalized involvement occurs elsewhere on
the body, but infection spares the mucous membranes. Despite the dramatic clinical picture, the entire process usually subsides with superficial

desquamation, and healing is usually complete within 5-7 days.

Toxic shock syndrome (TSS) (Choice E) is an acute febrile illness characterized by a generalized erythematous eruption accompanied by systemic
involvement. Toxic shock syndrome is due to toxin-producing strains of Staphylococcus aureus. Massive cytokine release as a result of
toxin/superantigen activity is postulated to be the mediator of the clinical signs of toxic shock syndrome. Fever, rash, hypotension, and multiple organ
involvement are the hallmarks of toxic shock syndrome. Desquamation of the palms and soles, as seen in many bacterial toxin-mediated disorders,
usually follows the onset of the illness by 1-2 weeks. Clinical finding include the following:

The eruption of toxic shock syndrome is defined as diffuse macular erythroderma; however, a scarlatiniform eruption, often with flexural
accentuation, frequently is present.

Erythema and edema of palms and soles

Hyperemia of conjunctiva and mucous membranes

Strawberry tongue

Delayed desquamation of palms and soles

Multisystem organ involvement in toxic shock syndrome

Fever greater than 38.9C (102F)

Cardiovascular - Hypotension (systolic blood pressure <90 mm Hg), cardiomyopathy

Gastrointestinal - Nausea, vomiting, diarrhea (usually secretory)

Muscular - Rhabdomyolysis, severe myalgias, muscle tenderness, muscle weakness

Renal - Azotemia, acute renal failure (serum creatinine value >2 times normal)

Neurologic - Toxic encephalopathy probably related to cerebral edema

Pulmonary - Adult respiratory distress syndrome

Hepatic - Elevated serum aspartate aminotransferase and serum bilirubin levels (>2 times normal), centrilobular hepatic necrosis

Hematologic - Thrombocytopenia (<100,000/L), leukocytosis, disseminated intravascular coagulation

Metabolic - Hypophosphatemia, hypocalcemia, electrolyte imbalances (especially metabolic acidosis)

A 46-year-old male was admitted to the hospital for evaluation of proteinuria, non-specific cardiac symptoms, and a possible vasculopathy. He had
reddish macular-papular skin lesions since the age of 13. In between the papules were reddish macular lesions.
As a youth, he experienced acroparesthesias, joint pains, and did not perspire in hot weather. There was no known family history of skin lesions.
He had diffuse red to blue papules in the gluteal, lower abdominal and inguinal regions, and on the scrotum and penis, navel and the nipples; less
numerous lesions were also present in the armpits and on the flexor aspects of the thighs. Lesions were also seen on his buccal mucosa, upper lip, and
his palms and soles.
Routine hematological tests were normal
Renal function was normal except for mild proteinuria, up to 0.46 g/L
Electrocardiography disclosed a sinusoidal rhythm and concentric hypertrophy of the left ventricle. Echography also showed concentric hypertrophy of
the left ventricle and normal systolic activity; the intraventricular septum was thickened and had an unusual granular structure.
Ophthalmologic investigation revealed aneurysmal dilatations of the conjunctival vessels, and diffuse corneal opacities (i.e., corneal verticillata).
What further testing should be ordered for a definitive diagnosis?
1)
2)
3)
4)
5)

Doppler test
Assay for phenylalanine hydroxylase (PAH)
Measurement of alpha-gal activity
Dihydropterine reductase activity on blood spot
Sequence analysis of the tumor necrosis factor receptor superfamily 1A (TNFRSF1A) gene

The correct answer is choice C.

The history, clinical symptoms and the evolution of the disorder were consistent with the diagnosis of angiokeratoma corporis diffusum (Fabry disease).
Biochemical confirmation of the clinical and histologic diagnoses of Fabry disease is made by determining the leukocyte -Gal A activity, which in this
case was totally deficient.

Subsequently, the patients -Gal A mutation was identified as a missense mutation, N272S, the substitution of a serine for an asparagine residue at
amino acid 272 in the -Gal A polypeptide.

Fabry disease is an X-linked lysosomal storage disease that is caused by deficient activity of lysosomal enzyme -galactosidase A ( -Gal A). Most males
with no -Gal A activity develop the classic phenotype of Fabry disease, which affects multiple organ systems. The first clinical manifestations of the

disease, which consist of episodes of severe pain in the extremities (acroparesthesias), hypohidrosis, corneal and lenticular changes, and skin lesions
(angiokeratoma) develop in childhood.

In classically affected males, the symptoms usually become manifest in childhood, with patients typically presenting with acroparesthesias, burning
tingling pains in the upper and lower extremities. They also experience episodic attacks of excruciating pain, known as Fabry crises, which are triggered
by fevers, exercise, stress and/ or dramatic changes in the weather. Decreased sweating (hypohidrosis) or even anhydrosis is also present as are the
typical skin lesions (angiokeratomas) typically present on the abdomen, buttocks, flanks, penis, and scrotum.

Young patients also have corneal opacities, postprandial abdominal cramping, diarrhea, left ventricular hypertrophy and proteinuria. With advancing
age, the progressive lysosomal GL-3 accumulation, particularly in the vascular endothelium, leads to renal failure, vascular disease of the heart and the
brain, and premature demise in the fourth and fifth decades of life. In the absence of a previously diagnosed family member, the disease is often
recognized only after patients experience the severe late complications of the disease including cardiac and renal failure.

A missed diagnosis of Fabry disease without the opportunity for treatment could have had tragic consequences for a patient. Enzyme replacement
therapy (ERT), which has been shown to clear globotriaosylceramide from the kidneys, heart, and skin, is now the standard of care in the treatment of
symptomatic Fabry disease.
Doppler test (choice A) is used for detection of blood clots, in this though it may be used for screening of cardiac abnormalities, it is not a test for
definitive diagnosis.

Phenylketonuria (PKU) is an autosomal recessive metabolic genetic disorder characterized by a deficiency in the hepatic enzyme phenylalanine
hydroxylase (PAH) (choice B).This enzyme is necessary to metabolize the amino acid phenylalanine ('Phe') to the amino acid tyrosine. When PAH is
deficient, phenylalanine accumulates and is converted into phenylpyruvate (also known as phenylketone), which is detected in the urine.

Dihydropteridine reductase (DHPR) (choice D) deficiency, an autosomal recessive genetic disorder, is one of the causes of malignant
hyperphenylalaninemia due to tetrahydrobiopterine deficiency. Not only does tetrahydrobiopterin deficiency cause hyperphenylalaninemia, it is also
responsible for defective neurotransmission of monoamines because of malfunctioning tyrosine and tryptophan hydroxylases, both tetrahydrobiopterindependent hydroxylases. DHPR deficiency should be suspected in all infants with a positive neonatal screening test for phenylketonuria, especially when
hyperphenylalaninemia is moderate.

The autosomal dominant disease TRAPS is characterized by episodes of fever, myalgia, arthralgia or arthritis, rash, abdominal pain, pleuritis, periorbital
edema, and conjunctivitis lasting for at least 5 days.

Mutations of the TNFRSF1A receptor gene on chromosome 12 (choice E) have been recognized as the cause of TRAPS5. The disease mechanism is
commonly attributed to impaired TNFRSF1A receptor shedding after engagement and signaling by TNF-, which leads to a decreased amount of
receptor available to bind soluble TNF- and repress inflammation. Receptor shedding is normal with the R92Q mutation, which suggests that defective
receptor shedding does not account for the entire disease mechanism.

Some diseases develop manifestations that are generally considered to be pathognomonic. Several other designations are used to describe the
cutaneous manifestations of skin diseases including those associated with dermatomyositis, which is usually divided into pathognomonic, characteristic,
compatible, less common and rare.
Gottron's papules and Gottron's sign are both seen in dermatomyositis where they are usually considered to be classified as which of the following:
1)
2)
3)
4)
5)

Characteristic
Pathognomonic
Compatible
Less Common
Rare

The correct answer is choice B

Cutaneous manifestations of dermatomyositis are generally grouped as pathognomonic, characteristic, compatible, less common and rare.

The primary lesion is a symmetric, violaceous, macular erythema. As the diseease progresses it may develop a poikilodermatous atrophic appearance
with telangiectasia and pigmentary changes. Due to deposition of mucin, induration commonly then follows.

Pathognomonic manifestations include Gottron's papules and Gottron's sign. Gottron's papules, violaceous papules overlying the dorsal interphalangeal
or metacarpophalangeal areas, elbow or knee joints, occur in approximately 70 percent of patients with dermatomyositis. Gottron's sign is erythematous
or violaceous, often atrophic, macules or plaques in the same symmetric distribution pattern but sparing the interphalangeal spacesjust the opposite

dermatologic distribution pattern on the hand that is observed in patients with systemic lupus erythematosus.

Current classification does not include any other manifestations of dermatomyositis considered to be pathognomonic.

Dermatomyositis is a rare idiopathic inflammatory myopathy that also demonstrates skin lesions. Only one to 10 cases per million are found in adults
and one to 3.2 in children.

Early recognition and treatment are important ways to decrease the morbidity of systemic complications. Patient management includes careful
evaluation for underlying malignancy, physical therapy, antihistamines, sunscreen and oral corticosteroids. Poor prognosis is indicated when the disease
becomes poorly responsive, if there is a delay in diagnosis or if malignancy is present.The therapeutic goal is to maintain function and prevent or
minimize sequelae.

Polymyositis includes the inflammatory myopathy without the cutaneous findings. Although the etiology of dermatomyositis is still unknown, some
studies have reported an association with histocompatability antigens, environmental agents (e.g., virus, drugs) and autoimmunity.

The average age at diagnosis is 40. Almost twice as many women are affected as men.

The average age of onset in juvenile dermatomyositis is between five and 14 years. This subgroup of patients has a better prognosis than adult
patients. Modern therapy has reduced mortality from near 50 percent to less than 10 percent.

A 60-year-old man presents with progressive fatigue, dyspnea on exertion, and headaches over the past 8 months. He also reports intense pruritus after
he takes a warm shower. On physical examination, splenomegaly is noted. Results of laboratory studies reveal: hematocrit, 61% (normal, 39%-54%);
hemoglobin, 19.5 mg/dL (normal [male], 14-18 mg/dL); leukocyte count, 18 103/L (normal, 4.5 11 - 103/L]; and platelet count, 554 103/L
(normal, 150-450 103/L).

What is this patients most likely diagnosis?


1)
2)
3)
4)
5)

Chronic myelogenous leukemia (CML)


Essential thrombocytosis
Myelofibrosis
Polycythemia vera
Multiple Myeloma

The correct answer is choice D

Polycythemia vera is a slowly developing blood dyscrasia characterized by blood cell overproduction. While the condition, which is usually seen in adults
60 and over, is more likely to occur in males, can be fatal and affect all blood cell types. It results from red blood cell overproduction that result in this
conditions symptoms which include headache, dizziness, itchinessespecially following a warm bath or shower, skin erythema, dyspnea, limb
dysesthesias or weakness, chest pain, sense of abdominal fullness or bloating due to splenic enlargement, and fatigue. The goal of polycythemia vera
treatment is reduction of cell number and includes phlebotomy, medications including hydroxyuria and anagrelide, aspirin to reduce the likelihood of
thromboembolic event, and antihistamines for pruritis.

Chronic myelogenous leukemia (Choice a) is incorrect because this condition, known as CML, is a blood disorder characterized solely by overproduction
of immature polymorphonucleocytes. Symptoms of CML, which include chills, diaphoresis, fever, and fatigue, are due to organ invasion by and
dysfunction of the cancerous cells. While chronic myelogenous leukemia, which is seen more commonly in the elderly and has a slight predilection
towards males, is not considered a fatal condition, it usually deteriorates into acute myelogenous leukemia (AML), which possesses significant morbidity
and mortality concerns.

Essential thrombocytosis (Choice b) is incorrect because this condition, also known as primary thrombocytopenia, is a myeloproliferative disorder that
only involves platelets. It is characterized by megakaryocyte overproduction that results in platelet numbers in excess of 450,000/ul and is more
commonly seen in patients greater than age 60. Essential thrombocytosis symptoms and clinical findings include weakness, headaches, paresthesias,
bleeding, splenomegaly, and erythromelalgia (burning pain and erythema of the extremities) with digital ischemia. Essential thrombosis, which is treated
with aspirin, has a generally benign course if thrombotic concerns are avoided.

Myelodysplasia (Choice c) is incorrect because this condition, which is characterized by scarring of the bone marrow, results in anemia. Symptoms of
myelodysplasia include fatigue, dyspnea, abdominal fullness, left upper quadrant pressure or pain, easy bruising, easy bleeding, night sweats, fever,

recurrent infection, and bone pain. Myelodysplasia is more common after age 50 and can degenerate into leukemia. The goal of treatment is to increase
red cell numbers and includes blood transfusion, androgen therapy, chemotherapy, radiation therapy, splenectomy, thalidomide with steroids, and stem
cell transplantation.

A mother brings her 5 year old son to the doctor because he has been scratching an area on his chest for the past week. The itching is much worse at
night. He has no known contact with poison ivy and no insect bites. The child has been afebrile and otherwise acting well. The patients mother has
eczema, and has used some of her own steroid cream on her sons rash for the past three days, which was not helpful. Physical exam reveals a
clustered area of small erythematous papules that are excoriated and tipped with blood crusts on his right upper chest extending laterally under his
right arm. There are several faint thin, brownish lines just under the surface of the skin. Several of the web spaces between his fingers also seem to
have a similar appearing rash, as do the bottom of his feet.
What is the most likely diagnosis in this patient?
1)
2)
3)
4)
5)

Contact dermatitis
Atopic dermatitis
Scabies
Molluscum contagiosum
Adverse effects of topical steroids

The correct answer is choice C.

This is scabies. This problem is caused by a small mite that prduces severe pruritis - out of proportion to the appearance of the rash. Small
erythematous papules with excoriations is classic, as is the description of lesions occurring in the web spaces of the fingers. The burrow (thin gray-red
lines under the skin) of the mite is pathognomonic when correctly identified, but it can be difficult to see, especially when excoriations or secondary
infection are present. The rash can also appear on the flexor aspect of the wrists, anterior and posterior axillary folds, skin around the nipples,
periumbilicaly, around the genital area and on the buttocks, thighs and feet, but the back and head are usually spared.

A couple of other teaching points

1) Diagnosis is based on history and physical exam focusing on the lesion distribution and appearance of the rash. It is possible to confirm the diagnosis
by performing skin scrapings, but this is not necessary and has a low sensitivity, so dont use skin scrapings to rule out scabies. In difficult cases, a

therapeutic trial of anti-scabies medication is often the best way to pin down the diagnosis.

2) The first-line treatment of scabies includes topical permethrin cream or oral ivermectin. Lindane cream can be used, but is second-line in the pediatric
population due to its possible neurotoxicity. The key to successful eradication is to treat all family members and close contacts at the same time. Fomite
transfer is not as much of a concern as it is for head lice, but precautions should be taken including bagging personal items for 3 days and then washing
them.

Contact dermatitis (choice A), a rash resulting from direct skin exposure to an allergen or an irritant, is almost always on the differential diagnosis for an
itchy rash. The rash is usually intensely pruritic and if allergic in nature, can occur up to two weeks after exposure. The rash itself is usually papular and
erythematous with indistinct margins, distributed along areas of exposure. Fluid collects in the epidermis causing vesicles and oozing. Remember that
the poison ivy/oak/sumac group usually occur in linear patterns from kids brushing against the plant. When trying to determine the cause, looking at the
distribution can be helpful (i.e. scalp involvement - think about hair products; face - think about cosmetics and aftershaves, etc).

Atopic dermatitis or eczema (choice B) is another common type of rash considered to be familial and and allergic in nature. Most eczema presents
before age 7 and often occurs as part of the allergic triad with asthma and allergic rhinitis. The rash usually is intensely pruritic, appearing as
erythematous patches with scaling, but can also be vesicular or papular at different stages. In children, eczema usually occurs on the face, scalp,
extremities, diaper area and trunk, while in adults it tends to occur on in the flexural areas. Eczema is a clinical diagnosis and is treated with topical
steroids and UV light therapy.

Molluscum contagiosum (choice D) is a common chronic localized infection caused by the pox virus. The rash appears as flesh colored, dome-shaped
umbilicated papules on any part of the body. It is spread by direct skin to skin contact, autoinoculation, and from fomites. Diagnosis is usually made by
the classic appearance, but histological exam can help. This infection is usually self-limited and will resolve over several months in the
immunocompetent individual. Treatment outside of the genital area is for cosmetic reasons only and generally not done; it includes curettage,
cryotherapy, and laser therapy.

Cutaneous effects of steroid use, such as skin atrophy and telangiectasias (choice E) is not the best answer. In general, it takes 2-3 weeks before this
would occur even with medium to high potency steroids. Areas where the skin is thin are the most likely to be damaged by chronic high potency topical
steroids. The atrophy can be reversible if therapy is terminated as soon as the changes are first noticed, otherwise the steroid induced changes can be
everlasting. This patient has not been using his moms steroid cream long enough to cause skin damage from even a high-potency topical steroid.

However, the patients mom should be told about the dangers of steroid creams and instructed to stop using the cream on her son.

A 72 year-old man presents with severe left thigh pain in the area of a rash that began approximately 10 days prior to presentation. A crusting, linear,
erythematous rash is present on his anterior thigh, with areas of dysesthesia around the rash. Healing herpes zoster is diagnosed.
All of the following are appropriate treatments at this time EXCEPT:
1)
2)
3)
4)
5)

Opiate analgesics
Famciclovir
Amitriptyline
Topical lidocaine
Gabapentin

The correct answer is choice B

Famciclovir (Famvir), acyclovir (Zovirax) and valaciclovir (Valtrex) are members of a group of antiviral compounds that are FDA-approved for the
treatment of acute herpes zoster, more commonly known as shingles, because of their demonstrated ability to (1) reduce the length and severity of the
episode and (2) reduce the likelihood that the patient will development postherpetic neuralgiaa chronic pain condition located along the affected
dermatome. The majority of prescriptions written for the treatment of herpes zoster are for Valtrex, dosed 1 g tid, and Famvir, dosed 500 mg bid,
because of their simpler dosing schedules. Unfortunately, these medications possess a short window of therapeutic opportunity of only up to about 5
days after onset of symptoms and do not result in clinical improvement or decreased likelihood of preventing zoster complications if instituted once the
lesions have resolved.

Opiate analgesics (Choice A) is incorrect because herpes zoster can be very painful in both its acute and chronic stages.

Amitryptiline (Choice C), brand name Elavil, is incorrect because it is often helpful for patients suffering from postherpetic neuralgia and other types of
chronic pain.

Topical lidocaine (Choice D), brand name Lidoderm, is helpful in many types of neuropathic and musculoskeletal pain conditions, and is a reasonable

component of a pain management plan for either acute herpes zoster or postherpetic neuralgia.

Gabapentin (Choice E), brand name Neurontin, is incorrect because it is FDA-approved for the treatment of postherpetic neuralgia.

Figure 1: Acute treatment of HSV review

A 17 year old girl presents with pruritic eruption that consists of papules and vesicles localized to the wrist. Papules and vesicles are noted in a bandlike
pattern, with slight oozing form some lesions. She is other wise health and ho significant past medical history. She is on birth control pills and is a 1pack per day smoker.
The most likely cause of the rash is which of the following:

1)
2)
3)
4)
5)

Herpes Simplex
Shingles
Allergic Contact Dermatitis
Seborrheic Dermatitis
Psoriasis

The correct answer is Choice C.

Allergic contact dermatitis is inflammation of the skin manifested by varying degrees of erythema, edema, and vesiculation. It is a delayed type of
induced sensitivity (allergy) resulting from cutaneous contact with a specific allergen to which the patient has developed a specific sensitivity. The
prevalence of contact dermatitis to be 13.6 cases per 1000 population using physical examinations by dermatologists of a selected sample of patients.
Causes of the allergic contact dermatitis comprises lots of agents, however nickel is the leading cause of allergic contact dermatitis in the world.

The incidence of nickel allergic contact dermatitis in North America is increasing. Allergic contact dermatitis to nickel typically is manifested by dermatitis
at the sites where bracelet, earrings or necklaces containing nickel are worn. Acute allergic contact dermatitis is characterized by pruritic papules and
vesicles on an erythematous base. Lichenified pruritic plaques may manifest chronic allergic contact dermatitis.

Herpes simplex virus (HSV) infection (Choice A) may "typically" present with grouped vesicles arising on an erythematous base on keratinized skin or
mucous membrane. Most HSV infections are "atypical," with patch(es) of erythema, small erosions, fissures, or subclinical lesions that shed HSV. Once
an individual is infected, HSV persists in sensory ganglia for the life of the patient, recurring with lessening in immunity. In healthy individuals, recurrent
infections are asymptomatic or minor, resolving spontaneously or with antiviral therapy. In the immunocompromised host, mucocutaneous lesions can
be extensive, chronic, or disseminate to skin or viscera. Bandlike pattern around the wrist is not consistent with the HSV infection

Herpes zoster (shingles) (Choice B) is an acute cutaneous viral infection caused by the reactivation of varicella-zoster virus (VZV), a herpes virus that
initially produces chickenpox. Difference in clinical manifestations between varicella and zoster apparently depends on the immune status of individual
patients; those with no prior immunologic exposure to varicella virus, most commonly children, develop the clinical syndrome of varicella, while those
with circulating varicella antibodies develop a localized recrudescence, zoster. Classic physical findings of zoster include painful grouped herpetiform
vesicles on an erythematous base confined to the cutaneous surface innervated by a single unilateral sensory nerve. Regional lymphadenopathy may be
present. Vesicles initially are clear but eventually cloud, rupture, crust, and involute.

(Choice D) Seborrheic dermatitis (SD) is a very common chronic dermatosis characterized by redness and scaling and occurring in regions where the
sebaceous glands are most active, such as the face (central forehead, glabella, eyebrows, nasolabial folds) and scalp ("cradle cap"), the presternal area,
and in the body folds. The rash is represented by greasy, salmon-colored patches. Malassezia furfur is said to play a role in the pathogenesis, and the
response to topical ketoconazole and selenium sulfide is some indication that this yeast may be pathogenic, however the KOH preparation is often
negative.

Psoriasis (Choice E) is a systemic inflammatory disorder that manifests as sharply demarcated red plaques with silvery-white scales often on the
extensor surfaces and scalp. The Psoriatic Triad:

Lesions of bright pink or a red color, covered with abundant silvery white scales. Lesions have pronounced edges. With scraping the scales easily
separate from the lesions;

Under the separated scales there appears a reddish moist tender semi-transparent skin;

With further scraping at the moist tender skin there will appear small droplets of blood (point hemorrhage).

Clinical picture is not consistent with Choice A, Choice B, Choice D or Choice E.

You are a senior resident in pediatrics who is examining the 7-year-old boy who complains of perianal pruritus worsening at night. During the
examination you note excoriations and erythema of the perineum. Besides the excoriations and erythema there are no other findings.The child is doing
well and vitals are within normal limits. Development history is normal and immunizations are up to date.
The most likely diagnosis includes which of the following:
1)
2)
3)
4)
5)

Psoriasis
Enterobiasis
Crohn's disease
perianal abscess
None of the above

The correct answer is Choice B

Enterobiasis is a helmintic infestation caused by nematode E vermicularis. Prevalence of Enterobiasis is approximately 5-15% in the general population
and greatest in children aged 5-9 years. However, all ages can be affected. Patients with enterobiasis can be symptomatic or asymptomatic. If patients
are symptomatic, pruritus ani and pruritus vulvae are common presenting symptoms. Patients often have excoriation or erythema of the perineum,
vulvae, or both, but infestation can occur without these signs. Restless sleeping may be due to pruritus ani (Choice B). Infestation can cause enuresis.
Secondary bacterial skin infection may develop from vigorous scratching to relieve pruritus. Reinfestation is common. Infection can develop as long as
female pinworms continue to lay eggs on the skin.Worms may be incidentally discovered when they are seen in the perineal region.

Psoriasis (Choice A) is a systemic inflammatory disorder that manifests as sharply demarcated red plaques with silvery-white scales on the extensor
surfaces and scalp.
Psoriatic Triad:

Lesions of bright pink or a red color, covered with abundant silvery white scales. Lesions have pronounced edges. With scraping the scales easily
separate from the lesions;

Under the separated scales there appears a reddish moist tender semi-transparent skin;

With further scraping at the moist tender skin there will appear small droplets of blood (point hemorrhage).

Perianal pruritas at night is not typical for psoriasis.

Crohns disease (Choice C) is an iflammatory bowel disease. It may involve any portion of the GI tract from the mouth to the anus, particularly the
ileocecal region, in a discontinuous pattern (skip lesions) The rectum is often spared. Transmural inflammation is seen on a biopsy. Crohns disease
can cause abdominal pain, diarrhea, malabsorbption, bloody stools and fistula formation between the intestines and other organs.The clinical picture of

the Crohns disease including perianal pruritus at night is not consistent with the clinical case that excludes the Choice C.

Perianal abscess (Choice D) is the abscess of the skin around the anus. It usually results from obstruction of intestinal crypts, local bacterial invasion
and subsequent fistula formation in the skin. Males have perianal abscesses more often (4:1). It represented by localized induration and fluctulance due
to inflammation of the soft tissue near the rectum, perirectal throbbing pain, fever, and pain with defecation. The clinical picture does not coincide with
the clinical case.

(Choice E) All of the above is a wrong answer too.

A 28-year old man presents at your clinic with a large mole on the left foot with an irregular and indistinct border. A biopsy taken from a mole shows
cytologic atypia of melanocytes and diffuse epidermal infiltration by anaplastic cells which are also present in the papillary and reticular dermis.
What is the most likely diagnosis?
1)
2)
3)
4)
5)

Melanoma, Clark level IV


Congenital melanocytic nevus
Dysplastic nevus
Melanoma, Clark level III
Balloon cell nevus

The correct answer is Choice C.

Histologically, dysplastic nevi (Choice C) have melanocytic proliferation at the epidermodermal junction with some cytologic atypia.

People with dysplastic nevi should be routinely evaluated by a dermatologist. Removing all of the moles is neither practical nor recommended, nor does
it entirely remove the risk for developing melanoma. Dysplastic moles with an unusual appearance or suspicious changes are often removed for
microscopic evaluation. People at higher risk for melanoma should minimize exposure to other risk factors associated with melanoma.

In melanomas, Clark level IV (Choice A), the cancer has spread from the epidermis through the upper dermis and involves the lower dermis.

Congenital melanocytic nevus (Choice B) is found in infants at birth.

For melanomas, Clark level III (Choice D), the cancer involves most of the upper dermis. The appearance of the congenital melanocytic nevus is similar
to those of acquired ones, but are usually larger in diameter and may have excess hair, called hypertrichosis. The neval cells are also found deeper into
the dermis in congential melanocytic nevus.

Balloon cell nevus (Choice E) is a cutaneous condition characterized histologically by large, vacuolated nevus cells with clear cytoplasm and central
nucleus. The nucleus appears hyperchromatic. Multinucleate balloon cells may be present.

A four-year-old child presents to the ER with a consistent fever for five days, bilateral conjunctival injection without exudate, a polymorphous exanthem,
cervical lymphadenopathy, and dry, cracked, cherry red lips. On examination, the child has a temperature of 40 C (104 F), heart rate of 125/min,
respirarory rate of 26/min and a blood pressure of 90/60 mm Hg. The child is irritable but is alert. Past medical history consists of prior febrile seizures
which has resolved and recurrent otitis medical that is being treated with antibiotics.
Which of the following complications is most likely to occur?

1)
2)
3)
4)
5)

Coronary artery aneurysm


Gastrointestinal hemorrhage
Hepatic necrosis
Interstitial pneumonitis
Renal tubular acidosis

The correct answer is Choice A

Kawasaki disease (mucocutaneous lymph node syndrome) is a multisystemic acute large- and medium-vessel vasculitis that primarily affects young
children (80% are < 5 years of age) with predilection for those of Asian ancestry. Epidemics occur primarily in late winter and spring with 3-year
intervals, lending some credence to the possibility of an infectious etiology. Kawasaki disease is most commonly observed in children from the middle
and upper-middle classes.

The estimated number of children hospitalized annually in the United States is about 3000, though more than 4000 admissions occurred in 2006, some
of which were incomplete cases. The highest incidence in the United States is in Hawaii at 17 per 100,000 children younger than 5 years. Outside the
United States, the disease is most frequently observed in Japan, Taiwan, and Korea. The prevalence of Kawasaki disease increased from 1967 to the
mid 1980s and has leveled out at 5000-6000 cases per year. This condition is divided into acute, subacute, and chronic phases. Diagnostic criteria
include the following:

Lasts 12 weeks and presents with the following symptoms:

Fever persiting (usually > 40C (104 F) for at least five days AND 4 of the criteria below:

Bilateral, nonexudative, painless conjunctivitis.

Polymorphous rash (primarily truncal).

Cervical lymphadenopathy (often unilateral, with at least one node >1.5 cm).

Diffuse mucous membrane erythema (e.g., strawberry tongue, red fissured lips).

Erythema of the palms and soles; indurative edema of the hands and feet; late desquamation of the fingertips (in subacute phase).

Kawasaki disease can cause several serious complications that determine the degree of disability. One of the most dangerous complications of the
Kawasaki disease is the development of coronary artery aneurysms (CAA) (Choice A) with consequences such as thrombosis or rupture. Acute
myocardial infacrction (MI) has been reported secondary to true coronary artery obstruction. These aneurysms tend to develop approximately 1-2
weeks after onset of symptoms.

Myocarditis, congestive heart failure, pericarditis with pericardial effusion, mitral or aortic insufficiency, and dysrhythmias may be observed early in the
disease. Decreased left ventricular function is present in approximately 50% of all patients with Kawasaki syndrome. Additionally, arthritis persists in
some children. Consequently, gastrointestinal hemorrhage (Choice B), hepatic necrosis (Choice C), interstitial pneumonitis (Choice D) and renal tubular

acidosis (Choice E) are not among frequent complications of the Kawasaki disease.

A 46-year-old, left-handed woman complains to her doctor that for the past 5 months she has noticed a gradually worsening tremor in her dominant
hand when raising a cup of tea in the morning. More recently, she has noticed that her handwriting has become smaller and she is losing the ability to
appreciate fine smells. She tires easily, finding it difficult at times to concentrate when undertaking long stretches of driving, and she often has mild
aching in her left foot. On examination, she has mild upper left extremity rest tremor, bradykinesia with fine motor tasks (more pronounced on the left
side), slight rigidity with activation of the left upper arm, and a mildly slow gait. She was diagnosed with early, mild Parkinson's disease. She remains
active in her current employment and enjoys her work.
Which treatment would you recommend?
1)
2)
3)
4)
5)

Delayed treatment with levodopa


A neuroprotective treatment
Anticholinergics
Coenzyme Q10
Deep brain stimulation

The correct answer is Choice A.

Treatment plans for Parkinson's should include management of both motor and non-motor manifestations and should address quality of life issues. It is
important that she is able to continue working and be helped to learn how to deal with her Parkinson's without becoming overwhelmed by the
diagnosis. Given her young age, it is generally thought prudent to delay treatment with levodopa (Choice A) to avoid the development of motor
complications. Young patients are particularly prone to levodopa-induced motor complications; therefore, other medications are used as long as they
adequately control symptoms.

Chronic levodopa treatment for Parkinson's disease patients is frequently associated with the development of motor complications such as end-of-dose
wearing-off and dyskinesia. Medical management includes manipulation of levodopa dosing to establish the optimum treatment schedule, improving
levodopa absorption, catechol-O-methyl transferaseinhibition, monoamine oxidase-B inhibition, dopaminergic agonists, amantadine, and continuous
dopaminergic infusions.

Given her diagnosis of early Parkinson's, a treatment with neuroprotective effects to slow the progression of the underlying disease would be desirable.

At this time, no treatment is proven to slow the progression of the underlying disease process (Choice B).

There is some indication that anticholinergics such as trihexyphenidyl (Choice C) may be useful as treatment of early, but since quality of evidence on
efficacy is reduced they are not first-choice treatments.

Coenzyme Q10 (Choice D) has been shown to have an effect on the symptoms of Parkinson's disease; however, it is unclear whether it actually slows
the disease or simply temporarily alleviates symptoms.

Deep brain stimulation (Choice E) is used in people with advanced Parkinson's for whom drug therapy is no longer sufficient.

A 40-year-old woman presents for evaluation of mild pruritus, which she has had for the past year. She does not drink alcohol and takes no
medications. Physical examination shows an enlarged, firm liver. Results of laboratory studies are: alkaline phosphatase, 736 U/L (normal, 70-230 U/L);
aspartate aminotransferase, 100 U/L (normal, 7-40 IU/L); alanine aminotransferase, 160 U/L (normal, 0-40 IU/L); and serum bilirubin, 1.5 mg/dL
(normal, 0.2-1.0 mg/dL). Hepatitis serologies are negative. Abdominal ultrasonography shows no focal hepatic lesions.
What is this patients most likely diagnosis?
1)
2)
3)
4)
5)

Autoimmune hepatitis
Laennecs cirrhosis
Nonalcoholic steatohepatitis
Primary biliary cirrhosis
Primary sclerosing cholangitis

The correct answer is Choice D

Primary biliary cirrhosis is a chronic inflammatory disease of the liver that leads to obstruction of bile ducts (cholestatic disease of the liver) and eventual
hepatic cirrhosis. This disorder is characterised histologically by chronic non-suppurative destruction of interlobular bile ducts leading to advanced
fibrosis, cirrhosis, and liver failure.This condition is predominantly seen in females (10:1 female to male ratio) with usual onset between the ages of 30
and 60. Although 50 to 60 percent of patients are asymptomatic at diagnosis, they will develop symptoms later. The precise aetiopathogenesis of

primary biliary cirrhosis remains unknown, although dysregulation of the immune system and genetic susceptibility both seem to be important.

PBC can be associated with arthralgia and other non-hepatic autoimmune diseases, such as Sjgrens syndrome, sicca syndrome, rheumatoid arthritis,
thyroiditis and scleroderma. PBC and rheumatoid arthritis (RA) have been suggested to coexist in 1.8 to 5.6% of patients with PBC, but data supporting
this association are scarce. Presenting symptoms may also include fatigue, malaise, pruritus and jaundice. Diagnosis is based upon biochemical tests,
compatible liver histology, and the presence of antimitochondrial antibodies (AMA) or diseasespe cific antinuclear antibodies. Between 90% and 95% of
people with antimitochondrial antibody in serum, at titres of 1/40 or greater, have primary biliary cirrhosis. Antinuclear antibody and smooth muscle
antibody arise in 35% and 66% of patients with primary biliary cirrhosis.

PBC slowly, but almost invariably, progresses to cirrhosis over 10 to 30 years, and the diagnosis is often made late in its natural his tory. The hydrophilic
bile acid ursodiol is effective in decreasing liver injury due to cholestasis and in retarding the progression of disease. Patients who develop
decompensated cirrhosis require liver trans plantation. Presently, 4 percent of liver transplants in the United States are done for PBC.

Autoimmune hepatitis (Choice A) is not correct because this condition, a disease characterized by autoimmune hepatic assault, while it does occur more
frequently in women and does have similar presenting symptoms to primary biliary cirrhosis, is more likely to occur in younger women and in persons
with other autoimmune concerns.

Laennecs cirrhosis (Choice B) is incorrect because this is the term for liver damage resulting from chronic alcohol abuse. Our patient does not use
alcohol so this is eliminated as a possible cause of her hepatic concerns.

Nonalcoholic hepatic steatosis (Choice D) is incorrect because this is a condition of fatty infiltration of the liver that is essentially silent until the disease
is far progressed and tends to manifest with marked symptoms. Patients with this condition may or may not drink alcohol, it is seen in 2-4% of
Americans, and is suspected to be associated with high fat diets and obesity.

Primary sclerosing cholangitis (Choice E) is not correct because this condition occurs more frequently in men. The cause of primary sclerosing
cholangitis is usually not identifiable and patients often have autoimmune concerns or Crohns disease, sarcoidosis, chronic pancreatitis, or other chronic
medical issues.

A 38 year old male was hospitalized in the neurosurgery department for the monitoring of epileptic seizures due to a skull base fracture. On day 2 of
admission, he developed high fever, sloughing of total epidermis, and the clinical appearance of a severe burn patient.
The signs and symptoms presented upon clinical examination included:

fever
pain
erythema
positive Nikolsky sign
conjunctivitis
target-like lesions
blisters
mucous erosions
haemorrhagic crusting of the lips

The correct diagnosis of this patients condition is?


1)
2)
3)
4)
5)

Status epilepticus
Stevens-johnson syndrome (SJS)
Type IV hypersensitivity reaction
Pemphigus
Erythema multiforme (EM)

The correct answer is choice B.

The Stevens-Johnson syndrome (SJS) is a rare immune-complex-mediated hypersensitivity disorder which affects 7.1 per million persons. StevensJohnson syndrome (SJS) was considered an extreme variant of Erythema multiforme (EM) (choice E) for many years, while toxic epidermal necrolysis
(TEN) was considered a different entity. However, in 1993, a group of medical experts proposed a consensus definition and classification of EM, SJS,
and TEN based on a photographic atlas and extent of body surface area involvement. According to the consensus definition, SJS was separated from the
EM spectrum and added to TEN.

Essentially SJS and TEN are considered severity variants of a single entity. The two spectra are now divided into (1) EM consisting of erythema minor

and major (EMM) and (2) SJS/TEN. SJS/TEN -

Widespread blisters predominant on the trunk and face, presenting with erythematous or pruritic macules and one or more mucous membrane erosions;
epidermal detachment is less than 10% TBSA for SJS and 30% or more for TEN. SJS presents in three different forms which reflect the same condition:
a mild form, called erythema multiforme (where <10% TBSA is affected), the main form (between 10 and 30%), and the severe form, called toxic
epidermal necrolysis) (TEN).

The most important clinical signs and symptoms of SJS are the following:
prodromal signs: 2-3 days of malaise, rash, fever, cough, arthralgia, myalgia, rhinitis, headache, anorexia, and nausea and vomiting, with or without
diarrhoea
conjunctivitis, usually occurring 1-3 days before the skin lesions appear
intense erythema, progressing rapidly to epidermolysis and ceasing in 2-3 days
blisters
mucous membrane erosion
haemorrhagic crusting of the lips
epidermal detachment
positive Nikolsky sign
target-like lesions
extreme pain
dehydration, which may lead to hypovolaemic shock and death
mimicking of the staphylococcal scalded skin syndrome (similar appearance, but blisters rise nearer the skins surface).

Stevens-Johnson etiology is mainly a reaction to medication. More than 80% of cases of SJS/TEN are drug-related and 95% of TEN cases.
Bibliographical references report various infections occurring prior to TEN (mostly viral). Several drugs have been identified during the last decade as a
triggering cause, as in this case it was most probably anti-convulsant drug, valproic acid.

Status epilepticus (SE)(choice A) is a common, life-threatening neurologic disorder. It is essentially an acute, prolonged epileptic crisis.
A type IV hypersensitive reaction (choice C) involves the cell-mediated branch of the immune system. Antigen activation of sensitized TH1 cells induces
release of various cytokines that cause macrophages to accumulate and become activated. The net effect of the activation of macrophages is to release
lytic enzymes that cause localized tissue damage.

Pemphigus (choice D) is a rare group of blistering autoimmune diseases that affect the skin and mucous membranes. In pemphigus, autoantibodies
form against desmoglein. When autoantibodies attack desmogleins, the cells become separated from each other and the epidermis becomes "unglued",
a phenomenon called acantholysis. This causes blisters that slough off and turn into sores. In some cases, these blisters can cover a significant area of
the skin. The term pemphigus refers to a group of autoimmune blistering diseases of the skin and mucous membranes characterized histologically by
intraepidermal blister and immunopathologically by the finding of in vivo bound and circulating immunoglobulin G (IgG) antibody directed against the
cell surface of keratinocytes.

A 36 year old man has a 4 week history of a generalized, intensely pruritic papular rash. He is otherwise well. His 9 year old son also has intense itching.
On examination, there is a generalized rash consisting of excoriated papules and vesicles on wrists, sides of fingers, arms and buttocks. No other
abnormalities are noted.

Which one of the following is the most likely diagnosis?


1)
2)
3)
4)
5)

pityriasis rosea
neurodermatitis
scabies
bullous impetigo
Papular urticaria

The correct answer is Choice C

Scabies is a skin infestation caused by an extremely small mite. Scabies infestations are associated with close living quarters, poor sanitation, shared
clothing, and careless hygiene. Distinguishing characteristics of scabies is severe pruritis, high likelihood of other family members or close personal
contacts with similar rash or symptoms, and location of papules and burrows (red, raised linear markings leading to papules that indicate pathway of
pathogen movement) in the webspaces of fingers and crural areas. Treatment is with severe laundering of contaminated clothing and bedding
accompanied by topical permethrin applied to the patient from the neck down and left on overnight. The permethrin is removed in the morning during
bathing. Warnings must be given not to repeat daily applications of permethrin due to neurotoxocity concerns.

Pityriasis rosea (Choice A) is incorrect because the location of this exanthema is fairly generalized throughout the torso and is often described as being
in a Christmas tree pattern. The majority of P. rosea cases are seen in children and tend to present in clusters due to the contagious nature of the
condition (viral origin is suspected). The majority of cases follow a brief viral prodrome and are self-resolving in 4-8 weeks without treatment.

Neurodematitis (Choice B) is incorrect because the lesions of neurodermatitis are self-inflected and result from digging at the skin by a person suffering
from psychological and psychiatric concerns. Characteristic excoriated lesions are seen on the head and neck, arms and torso. Treatment is a
combination of reduction of the mental health symptomatology along with observation for and treatment of lesions that can become superinfected.

Bullous impetigo (Choice D) is incorrect because this condition is almost exclusively seen in infants up to 2 years of age. One notable exception to this
rule is outbreaks of streptococcal or staphylococcal bullous impetigo amongst athletic teams. The lesions typical of this condition are bullae that tend to
be distributed on the torso of babies as opposed to the small red papules seen in the patient in our example.
A 70-year-old woman presents to her family physician requesting more information about the herpes zoster vaccine. Her friends have told her about the
new vaccine, and she wants to know if she is a candidate to receive it. She had chicken pox as a child but has not had shingles. She notes that she is
concerned about shingles because of the debilitating pain associated with this condition.
Use of the herpes zoster vaccine
1) Persons aged = 40
2) Persons aged = 50
3) Persons aged = 60
4) Persons aged = 70
5) Persons aged = 75

is recommended mainly in which of the following age-groups?


years
years
years
years
years

The correct answer is choice C

The following is a direct quote from the reference web site provided by the FDA concerning their licensing of the vaccine Zostavax:

"FDA has licensed Zostavax, a new vaccine that helps to reduce the risk of getting herpes zoster (shingles) in individuals 60 years of age and older.

Zostavax is the only US licensed vaccine that reduces the risk of reactivation of the varicella zoster virus, the same one that causes chicken pox, and
remains dormant in the body after recovering from this infection."

The incidence and severity of the disease increase with age with more than half being older than 60 years. Almost 50 percent of older persons will
develop complications.

Postherpetic neuralgia, a neuropathic pain syndrome that persists or develops after the rash has healed, is the most common debilitating complication.
Its frequency and severity also increase with age.

Pain and discomfort can be prolonged and disabling, diminishing the patients quality of life and ability to function to a degree comparable to congestive
heart failure, myocardial infarction, diabetes mellitus type 2, and major depression.

Antiviral therapy can reduce the severity and duration, but it fails to prevent postherpetic neuralgia, which may persist for years and is frequently
refractory to treatment.

A 3 year old child is presented with red, scaly rash involving the scalp, arms and legs. Previous application of over the counter hydrocortisone cream has
not cleared complitely the rash. The child is doing well and vitals are within normal limits. Development history is normal and immunization is up to
date. On examination the lesions are bright pink and a red color, covered with abundant silvery white scales. Lesions have pronounced edges. With
scraping the scales easily separate from the lesions and denude a reddish moist tender semi-transparent skin.With further scraping at the moist tender
skin there will appear small droplets of blood.

Which of the following is the most likely diagnosis?


1)
2)
3)
4)
5)

Psoriasis
Seborrheic Dermatitis
Seborrheic Keratosis
Keratosis Pilaris
Lupus Erythematosus

The correct answer is Choice A.

Psoriasis is a systemic inflammatory disorder that manifests as sharply demarcated red plaques with silvery-white scales often on the extensor surfaces
and scalp.

Psoriatic Triad:

Lesions of bright pink or a red color, covered with abundant silvery white scales. Lesions have pronounced edges. With scraping the scales easily
separate from the lesions;

Under the separated scales there appears a reddish moist tender semi-transparent skin;

With further scraping at the moist tender skin there will appear small droplets of blood (point hemorrhage).

(Choice B) Seborrheic dermatitis (SD) is a very common chronic dermatosis characterized by redness and scaling and occurring in regions where the
sebaceous glands are most active, such as the face (central forehead, glabella, eyebrows, nasolabial folds) and scalp ("cradle cap"), the presternal area,
and in the body folds. The rash is represented by greasy, salmon-colored patches. Malassezia furfur is said to play a role in the pathogenesis, and the
response to topical ketoconazole and selenium sulfide is some indication that this yeast may be pathogenic, however the KOH preparation is often
negative. Despite some resemblance with psoriasis, lack of Psoriatic Triad and less prominent infiltration of plaques allow to exclude the SD.

(Choice C) Seborrheic Keratosis is represented by sharply delineated often scaly hyperpigmented plaques with greasy srust that appear stuck on the
surface of the skin. Typically, it appears at persons older than 30. Age of 1-year-old and clinical picture exclude the Seborrheic Keratosis.

(Choice D) Keratosis pilaris (KP) is a disorder of hyperkeratosis. It is very common genetic follicular condition that is manifested by the appearance of
rough bumps on the skin. It most often appears on the back and outer sides of the upper arms, and can also occur on the thighs, hands, and tops of
legs, flanks, buttocks, or any body part except the palms or soles of feet). The clinical description is not consistent with the KP.

(Choice E) Lupus Erythematosus (LE) rash typically consists of well-demarcated, annular, erythematous, scalling plaques. Psoriatic Triad is also
negative.The clinical picture is not consistent with the description, so the LE can be excluded.

A 22-year-old woman is bothered by an erythematous rash involving her cheeks and nose every time she spends more than a few minutes outdoors on

a sunny day. On physical examination between these episodes, there are no abnormal findings. A biopsy of the skin involved with the rash shows
deposition of IgG along the basement membrane by immunofluorescence microscopy, but uninvolved skin does not demonstrate this IgG deposition.
Her antinuclear antibody test is negative. She has no other complaints. Her renal function tests are normal.
Which of the following is the most likely diagnosis?
1)
2)
3)
4)
5)

Discoid lupus erythematosus


Progressive systemic sclerosis
Contact dermatitis
Systemic lupus erythematosus
Dermatomyositis

The correct answer is choice A

Discoid lupus erythematosus DLE (choice A) is the correct answer. This disease presents with coin-like or disk shaped lesions on the skin. The lesions
are hyperkeratotic elevations displaying hypopigmented surfaces with hyperpigmented borders. During the early stages of DLE one develops small red
macules which leaves a scar, if it progresses it can develop into a large disfiguring plaque. This type of lupus is only present with cutaneous lesions and
a high level of photosensitivity, no immunological abnormalities. The rash can be of two presentations localized or generalized. The localized form
occurs mainly on the face, scalp,ear, and base of neck. The generalized type of DLE can appear anywhere, normally somewhere below the neck, on the
arms, forearms and hands, with the palms and soles being the most painful areas of rash eruptions. Both diseases can progress into full blown lupus
(SLE) with the highest propensity being in generalized DLE. About 20% of DLE patients develop SLE; comparatively, about 25% of SLE patients develop
DLE. Fortunately, when a SLE patient develops DLE their lupus clinical picture is not as debilitating. Compared to a patient with only SLE and all the
immunological abnormalities without the skin lesions.

Progressive systemic sclerosis (PSS) (choice B) would be the best answer if the question dealt with a woman between the age of 20-55 years old, who
has fibroblast hyperplasia and deposition within the connective tissue or visceral organs. Another name for PSS is called scleroderma, it predominantly
deposits within the skin, GI , lungs, kidneys, heart, and skeletal muscles. Unfortunately, the clinical presentation depends on the organs involved. The
main complaints start off as Raynaud's phenomenon or polyarthritis, which progresses to pain and stiffness in the hands and in the fingers. Overtime,
the disease spreads from the upper part of the body to the lower regions with edema and stiffness in the periphery joints. Additionally, the face has a
tightening of the skin which appears similar to a mask without wrinkles. Remember, there are two types, diffuse of limited. Diffuse starts at the skin
level and spreads rapidly to internal organs; however, limited presents with CREST syndrome manifestations (calcinosis, Raynaud's phenomenon,
esophageal dymotility, sclerodactyly and telangiectasia) and gradually moves towards visceral organs.

Contact dermatitis (choice C) is the correct answer if the question dealt with type 4 delayed hypersensitivity skin presentations from the sun. Granted
elements of this question is similar to a person with an ultraviolet radiation photoimmunological sensitivity. The question does not focus on deficiencies
found in antigen presenting cells or in specific cytokines being released during sun exposure to further support this as being the correct answer. Since a
complete understanding of such a mechanism is not well understood, outside of the scope of cancer formation, this is not the right answer.

Systemic Lupus erythematous SLE (choice D) is the correct answer when the question deals with multiple organs being diseased. One would also see a
combination of skin lesions, joint pain, muscle pain, weight loss, fever and malaise. In addition, blood samples may present positive for anti-DNA and
anti Smith antibodies. Furthermore, antinuclear antibodies may also be found, but they are not specific to SLE.

Dermatomyositis (DM) (choice E) is incorrect. Keep in mind, DM should be included in the differential diagnosis because of the rash over the base of the
eyelids and base of neck. However, if it was the correct answer two things must be ruled out to make this choice . First of all, does the patient
complain about joint pain more than muscle pain, as in DLE and SLE, no. Dematomyositis is a inflammatory muscle disease with autoimmunity against
tRNA synthetase or more specifically; Jo-1 antibodies found in serum samples. Secondly, is there a decrease in WBC, and an increase in globulin as in
SLE and DLE; if so, the answer is not DM. Women presenting with DM should be checked for age appropriate visceral cancers (i.e. lungs, ovaries, and
stomach). Finally, elevated levels of aldolase and creatine kinase can be an effective tool, after muscle biopsy, of course; when diagnosing DM.

A 2 month old girl is brought into your office for a well child check. Her past medical history is remarkable only for two episodes of croup, and she has
been otherwise well. Physical examination is positive for a 2 cm, raised, well-demarcated area of erythema on her left cheek in the region of the angle
of the mandible. Her mom reports that she has had this mark since she was born and that it has not changed in size. She has no other lesions on her
body.
What is the next best step in management of this patient?
1)
2)
3)
4)
5)

Observation
laser therapy
oral prednisone
direct laryngoscopy
neck radiographs

The correct answer is D.

The patient described in the vignette has a hemangioma. Hemangiomas are very common in infancy, occurring in up to 2% of newborns. They are
benign vascular hamartomas and usually manifest by 2 months of age. The most common areas affected are the face, back, and chest, but they can
occur anywhere on the skin as well as on internal structures. For most hemangiomas, no treatment is necessary as they will spontaneously involute as
the child grows older. The vast majority involute by 9 years of age, with up to 60% involuting by 5 years of age.

Hemangiomas can become symptomatic depending on their location. When hemangiomas are found in the beard distribution area on the face and neck,
the presence of a congenital subglottic hemangioma must be considered. A key element in this baby's history is the recurrent "croup." Congenital
subglottic hemangiomas often present with a hoarse, barking, croup-like cough and stridor that is more prominent in the expiratory phase. As many as
50% of congenital subglottic hemangiomas present with cutaneous hemangiomas in the beard distribution, as in this patient.

Congenital subglottic hemangiomas are best diagnosed by direct laryngoscopy and visualization of the hemangioma. Prompt diagnosis is necessary, as
the hemangioma can grow rapidly and impinge upon the airway. Once diagnosed, treatment may involve a course of oral steroids, intralesional steroid
injections, or endoscopic laser ablation, among other options.

Observation (choice A) would be an appropriate choice if the hemangioma was not located in the beard distribution or if the baby had not had
symptoms of a congenital subglottic hemangioma (croup-like cough). One must rule out a congenital subglottic hemangioma in this situation.

Laser therapy (choice B) is rarely required for cutaneous lesions and is performed only if the hemangioma is sufficiently large to be grossly disfiguring or
impinging on other vital structures. If congenital subglottic hemangioma is diagnosed, laser ablation of that lesion, however, may be an option.

Steroids (choice C), both oral and intravenous, for cutaneous lesions is rarely necessary except in the event of very large hemangiomas or those
impinging on other vital structures. As with the above answer, if congenital subglottic hemangioma is diagnosed, a course of steroids for treatment of
that lesion may be recommended.

Neck radiographs (choice E) would be expected to show an asymmetric subglottic narrowing of the region affected by the hemangioma, if present.
Definitive diagnosis, however, is made by direct laryngoscopy. In this vignette, one has sufficient evidence to suspect congenital subglottic

hemangioma, and direct visualization of the lesion is the most appropriate next step.

30 year old female dentist presents with a soft smooth erythematous nodule on her lower lip. She states that a few weeks prior she had some chapped
lips with occasional bleeding. Now, the lips have healed but this new lesion arose suddenly in its place on her right small finger. It is occasionally tender
to pressure and has bleed in the past.

The most likely diagnosis includes which of the following:


1)
2)
3)
4)
5)

Herpetic whitlow
Cherry hemangioma
Pyogenic granuloma
Dermal nevus
Basal cell carcinoma

The correct answer is Choice C

Pyogenic granuloma, also known as lobular capillary hemangioma, is a common benign vascular lesion of the skin and mucosa of unknown etiology that
usually occurs in children and young adults. These rapidly developing lesions present as a solitary glistening red papule or nodule located on the head,
neck, extremities or torso and are prone to bleeding and ulceration. The cause of pyogenic granuloma is unknown. The following factors have been
identified as having a possible role to play in their development:

Trauma: some cases develop at the site of a recent minor injury, such as a pinprick.

Infection: Staphylococcus aureus is frequently present in the lesion

Hormonal influences: they occur in up to 5% of pregnancies and are rarely associated with oral contraceptives.

Drug-induced; multiple lesions sometimes develop in patients on systemic retinoids (acitretin or isotretinoin) or protease inhibitors

Viral infection is possible but not proven

Underlying microscopic blood vessel malformations

Lesions usually first appear as a small pinhead-sized red, brownish-red or blue-black spot that grows rapidly over a period of a few days to weeks to
anywhere between 2mm and 2cm in diameter. Occasionally they may reach up to 5cm. They bleed easily and may ulcerate and form crusted sores.
Usually a single lesion is present but in rare cases groups of multiple lesions may develop. Lesions most frequently appear on the head, neck, upper
trunk and hands (especially fingers) and feet. The pregnancy variant of pyogenic granuloma most often occurs on the mucosal surfaces inside the
mouth. Treatment most often consists of surgical excision, cryotherapy, and more recently, topical imiquimod cream and alitretinoin gel have both been
successfully used to treat pyogenic granulomas.

Herpetic whitlow (choice A) is an intensely painful infection of the hand involving 1 or more fingers that typically affects the terminal phalanx.1 Herpes
simplex virus 1 (HSV-1) is the cause in approximately 60% of cases of herpetic whitlow, and herpes simplex virus 2 (HSV-2) is the cause in the
remaining 40%.

Cherry Angioma (Choice B) is not correct because these reddish pinpoint papules or macules are the most common cutaneous vascular lesion seen in
adults. Usually appearing in the 30s and 40s, cherry angiomas are harmless, have equal gender distribution, and tend to increase in frequency with age.

Dermal nevus (Choice D) is incorrect because these raised extremely common lesions usually begin as brown or brown-black in color, but may convert
to fleshy or pearly color with time. They are not associated with trauma, develop slowly, and may occur anywhere on the body. Care must be taken to
distinguish these lesions from nodular melanoma.

A 9-month-old infant presents with a generalized eruption composed of multiple blanchable macules and papules that are 1-5 mm in diameter. The
mother tells you that the fever lasted approximately 3 days and subsided over the past 24 hours and then she developed this generalized eruption. The
diagnosis of Roseola infantum was made and the child subsequently treated.
Roseola infantum is caused by which of the following viruses:

1)
2)
3)
4)
5)

Human herpes virus 8 (HHV-8)


Human herpes virus 6 (HHV-6)
Parvovirus B19
Coxsackie virus
Epstein-Barr virus

The correct answer is Choice B

Roseola infantum is a common childhood disease that is caused by human herpesvirus 6 (HHV-6). The classic presentation of roseola infantum is a 9- to
12-month-old infant who acutely develops a high fever and often a febrile seizure. After 3 days, a rapid subsidence of fever to normal temperature
occurs, and a morbilliform rash appears. Similarly to other herpes viruses, HHV-6 remains latent in most patients who are immunocompetent. HHV-6 is
a major cause of morbidity and mortality in patients who are immunosuppressed. In the primary infection, replication of the virus occurs mainly in the

leukocytes and the salivary glands.

Serologic tests indicate that human herpesvirus 6 (HHV-6) infection is common among the population. In roseola infantum, age ranges from 2 weeks to
3 years.The HHV-6 has been reported to be responsible for 10-45% of cases of febrile illness in infants. The population-based study revealed primary
HHV-6 infection cumulative percentages of 40% by age 12 months and 77% by age 24 months.The peak age of acquisition of primary HHV-6 infection
is 9-21 months.

Despite the high fever, few clinical findings are observed early in the course of roseola infantum. The lack of upper respiratory tract infection is notable,
and meningeal signs and encephalopathy are not present. Gastrointestinal symptoms, signs of electrolyte imbalance, or evidence of dehydration are
rarely present. A febrile seizure, with no residual findings, may have occurred. After an abrupt loss of fever, the characteristic rash appears. The
eruption is generalized and composed of discrete, small, pale pink papules or a blanchable, maculopapular exanthem that is 1-5 mm in diameter. This
rash may last 2 days. The characteristic enanthem (Nagayama spots) consists of erythematous papules on the mucosa of the soft palate and the base
of the uvula. The enanthem may be present on the fourth day in two thirds of patients with roseola.

HHV-8 (Choice A) is considered to be a cause of Kaposi Sarcoma (KS) that is the lymphatic endothelial malignancy. The KS is composed of multiple red
or purple macules, papules, and nodules. Skin lesions firslty appear on the skin or mucous membranes, however may involve internal organs.

Parvovirus B19 (Choice C) infection is common in school-aged and younger children who attend daycare facilities. In general, children transmit the virus
to parents and siblings. In young children, the antibody seroprevalence ranges from 5-10%. This increases to 50% in adolescents and approaches 90%
in the elderly. Common symptoms of parvovirus B19 (B19V) infection include a mild nonspecific prodromal illness that may consist of fever (15-30% of
patients), malaise, headache, myalgia, nausea, and rhinorrhea; typically beginning 5-7 days after initial infection. These symptoms correspond to the
initial viremia and dissipate in 2-3 days. Approximately 1 week later, a bright red macular exanthem appears on the cheeks and is often associated with
circumoral pallor. A diffuse maculopapular rash (erythematous macules and papules that become confluent, giving a lacy or reticulated appearance) can
appear 1-4 days later and fades to a lacy erythematous rash, which may be pruritic and may spread gradually toward the distal extremities. Most
seropositive patients have no history of this classic biphasic illness. The clinical symptoms widely vary, and the classic "slapped cheek" rash is much
more common in young children. Often, Parvovirus B19 can cause self-limited arthralgias.

Coxsackieviruses (Choice D) can cause several ilnesses.Thus, group A coxsackieviruses tend to infect the skin and mucous membranes, causing
herpangina, acute hemorrhagic conjunctivitis (AHC), and hand-foot-and-mouth (HFM) disease. Group B coxsackieviruses tend to infect the heart, pleura,
pancreas, and liver, causing pleurodynia, myocarditis, pericarditis, and hepatitis. Both group A and group B coxsackieviruses can cause nonspecific

febrile illnesses, rashes, upper respiratory tract disease, and aseptic meningitis. Hand-foot-and-mouth disease (HFMD) is a systemic infection caused by
coxsackievirus A16, characterized by ulcerative oral lesions and a vesicular exanthem on the distal extremities in association with mild constitutional
symptoms. Herpangina is caused by coxsackievirus A16, 8, 10, 22; also coxsackie group B (strains 14). echoviruses, and other enteroviruses. It
usually affects children <5 years and is prevalent in late summer and early fall in temperate climates. The symptoms are a sudden onset of fever,
malaise, headache, anorexia, dysphagia, and sore throat. The case does not describe he symtom typical for Coxsackieviruses type A nor type B.

Epstein-Barr virus (EBV) (Choice E) causes Iinfectious mononucleosis that has been recognized as a clinical syndrome consisting of fever, pharyngitis,
and adenopathy. Most patients with infectious mononucleosis are asymptomatic and, therefore, have few if any symptoms. The incubation period of
EBV infectious mononucleosis is 1-2 months. The classic presentation of EBV infectious mononucleosis in children and young adults consists of the triad
of fever, pharyngitis, and lymphadenopathy. Early signs include fever, lymphadenopathy, pharyngitis, rash, and periorbital edema.

A 34 year old woman presents at your clinic complaining of attacks of "terrible dizziness" that she experienced while taking her two young children on a
merry-go-round several days ago. After questioning her, she reveals that what she experienced actually was a sudden sensation that she was spinning
faster and in a different direction than the merry-go-round was rotating. This occurred when she turned from one child to the other. She denies
experiencing similar episodes or motion sickness when she was younger. She is a non-smoker and denies consuming any alcohol prior to any of her
episodes of "dizziness".
At this point, you should include all of the following in your differential diagnosis, except:
1)
2)
3)
4)
5)

motion sickness
benign paroxysmal positional vertigo
labyrinthitis
Mnire disease
vestibular neuronitis

The correct answer is choice A.

The word "dizziness" could mean disequilibrium, presyncope, or vertigo, and the etiology can involve any of the systems listed. Disequilibrium is the
sensation that one is off-balance. It is not connected with nausea and can result of falls, often in a specific direction. Presyncope is lightheadedness or
the feeling that one is about to faint, while vertigo refers to a sensation of spinning or other movement and often is associated with nausea and
vomiting and nystagmus. Thus, it is important to question the patient carefully as to the exact sensations she's experienced and how they related to her
time on the merry-go-round.

Disturbances in the vestibular system can lead to vertigo, which often results from conflicting inputs from vestibular, proprioceptive, and visual systems.
If the problem is involves the brain, the term central vertigo is used, while peripheral vertigo refers to a disturbance involving the vestibular apparatus
(labyrinths) of the inner ear, or cranial nerve VIII. Compared to central vertigo, peripheral vertigo manifests with symptoms that are of shorter duration,
but with an abrupt onset and more severity, which sounds like what happened in the case of this woman. Associated nystagmus tends to be only
horizontal, not vertical.

Despite the nausea that patient experienced, motion sickness is unlikely. Motion sickness appearing for the first time during one's thirties is very rare.
Generally, it begins between the ages of 2-12 years and patients who experience it typically have a history of childhood carsickness. Add to this the fact
that the attacks were associated with specific head movements while the patient was in a rotating environment. Therefore, the nausea in this patient is
secondary to vertigo which occurred as a result of her head movements.

On a rapidly-rotating amusement ride, it is normal to develop vertigo with head movements. However, unless this was an unusually rapidly-rotating
merry-go-round, most people would not develop vertigo, unless some pathology is present. Given the abruptness of the symptoms, you should be
considering etiologies of peripheral vertigo, one of the most common of which is known as benign paroxysmal positional vertigo (choice B). This can
results dislodging of otoconia from their place in the utricle. Gradually they can migrate into one of the semicircular canals.

Labyrinthitis (choice C) can present with severe vertigo, nausea and vomiting, and may also include hearing loss. Typically, this results from an upper
respiratory infection. While vestibular neuronitis (choice E) refers specifically to the inflammation of the vestibular branch of cranial nerve VIII, since
generally it occurs in association with labyrinthitis the two words often are used synonymously.

Mnire disease (idiopathic endolymphatic hydrops, choice D) is thought to result from abnormalities of membranous labyrinth, due to an excess of
endolymph. It manifests with vertigo, tinnitus, and inconsistent sensorineural hearing loss. Although a reporting bias may be the reason for this, the
reported incidence is highest in Caucasians. Additionally, the incidence is slightly higher in females compared to males. Although peak occurs between
the ages of 40 to 60 years, the condition has been diagnosed in people as young as age four and also in elderly individuals.

A 10-year-old child has joints that show marked hypermobility, and her skin is hyperextensible, and, once released, it immediately returns to its original
state.Her height and weight are at the 50th percentile.

These features are most suggestive of which of the following inherited diseases?

1)
2)
3)
4)
5)

Marfan syndrome
Osteogenesis imperfecta
Ehlers-Danlos syndrome
Von Recklinghausen disease
Xeroderma pigmentosa

The correct answer is Choice C.

Ehlers-Danlos syndrome (EDS) (also known as "Cutis hyperelastica") is a group of inherited connective tissue disorders, caused by a defect in the
synthesis of collagen (a protein in connective tissue). . Depending on the individual mutation, the severity of the syndrome can vary from mild to lifethreatening. Currently,11 variants of Ehlers-Danlos syndrome are identified; all have genetic, biochemical, and clinical differences. The collagen defect
has been identified in only 6 of the 11 types of Ehlers-Danlos syndrome. Type IV is characterized by a decreased amount of type III collagen. Types V
and VI are characterized by deficiencies in hydroxylase and lysyl oxidase, an important posttranslational modifying enzyme in collagen biosynthesis.
Type VII has an amino-terminal procollagen peptidase deficiency. Type IX has abnormal copper metabolism. Type X has nonfunctioning plasma
fibronectin.

There is no cure, and treatment is supportive, including close monitoring of the digestive, excretory and particularly the cardiovascular systems.
Corrective surgery may help with some of the problems that may develop in certain types of EDS, although the condition means that extra caution is
advised and special practices observed. More than one third of persons with Ehlers-Danlos syndrome do not fit exactly into a single type; overlap is
common. Common to almost all groups is a unique appearance of the skin. The skin is usually white and soft, and underlying vessels are sometimes
visible.The skin has a doughy feel and easily hyperextensible. It is easy to pull, and, once released, it immediately returns to its original state.

Marfan syndrome (Choice A) is an inherited connective-tissue disorder transmitted as an autosomal dominant trait. Marfan syndrome results from
mutations in the fibrillin-1 (FBN1) gene on chromosome 15, which encodes for the glycoprotein fibrillin. Fibrillin is a major building block of microfibrils,
which constitute the structural components of the suspensory ligament of the lens and serve as substrates for elastin in the aorta and other connective
tissues. The diagnosis of Marfan syndrome remains mainly clinical. Affected patients are usually taller and thinner than their family members. Their
limbs are disproportionately long compared with the trunk (dolichostenomelia). Arachnodactyly is a common feature. Major criteria comprise pectus
excavatum or pectus carinatum, reduced upper-to-lower body segment ratio (0.85 vs 0.93). Arms and legs may be unusually long in proportion to the
torso. More than 60% of patients have scoliosis. The most serious complications are the defects of the heart valves and aorta. It may also affect the
lungs, eyes, the dural sac surrounding the spinal cord, skeleton and the hard palate.

Osteogenesis Imperfecta (OI) (Brittle Bone Disease, "Lobstein syndrome") (Choice B) is a genetic bone disorder. People with OI are born with defective
connective tissue, or without the ability to make it, usually because of a deficiency of Type-I collagen.

The following 4 types of osteogenesis imperfecta have been reported:

Type I - Mild forms

Type II - Extremely severe

Type III - Severe

Type IV - Undefined

Precise typing is often difficult. Severity ranges from mild forms to lethal forms in the perinatal period. Patients most commonly present with fractures
after minor trauma. In severe cases, prenatal screening ultrasonography performed during the second trimester may show bowing of long bones,
fractures, limb shortening, and decreased skull echogenicity. Lethal osteogenesis imperfecta cannot be diagnosed with certainty in utero. Often, patients
may bruise easily. Patients may have repeated fractures after mild trauma. However, these fractures heal readily. Deafness is another feature. About
50% of patients with type I osteogenesis imperfecta have deafness by age 40 years.

Neurofibromatosis (commonly abbreviated NF, also known as Von Recklinghausen disease) (Choice D) is a genetically-inherited disorder in which the
nerve tissue grows tumors (i.e., neurofibromas) that may be harmless or may cause serious damage by compressing nerves and other tissues. The
disorder affects all neural crest cells (Schwann cells, melanocytes, endoneurial fibroblasts). Cellular elements from these cell types proliferate excessively
throughout the body forming tumors and the melanocytes function abnormally resulting in disordered skin pigmentation. The tumors may cause bumps
under the skin, colored spots, skeletal problems, pressure on spinal nerve roots, and other neurological problems.

There are 2 types of NF:


Neurofibromatosis type 1 - mutation of neurofibromin chromosome 17q11.2 and The diagnosis of NF1 is made if any two of the following seven criteria
are met:

Two or more neurofibromas on the skin or under the skin or one plexiform neurofibroma (a large cluster of tumors involving multiple nerves);
Neurofibromas are the subcutaneous bumps that are characteristic of the disease and increase in number with age.

Freckling of the groin or the axilla (arm pit).

Caf au lait spots (pigmented, most often a shade of brown, smooth edges(coast of california)[2] birthmarks). Six or more measuring 5 mm in
greatest diameter in prepubertal individuals and over 15 mm in greatest diameter in postpubertal individuals

Skeletal abnormalities, such as sphenoid dysplasia or thinning of the cortex of the long bones of the body (i.e. bones of the leg, potentially
resulting in bowing of the legs)

Lisch nodules (hamartomas of iris), freckling in the iris.

Tumors on the optic nerve, also known as an optic glioma

Neurofibromatosis type 2 - mutation of NF2 (Merlin) in chromosome 22q12 and for this type, the diagnostic criteria include the presence of:

bilateral tumors, acoustic neuromas on the vestibulocochlear nerve (the eighth cranial nerve) leading to hearing loss

In fact, the hallmark of NF 2 is hearing loss due to acoustic neuromas around the age of twenty

the tumors may cause:

headache

balance problems, and Vertigo

facial weakness/paralysis

patients with NF2 may also develop other brain tumors, as well as spinal tumors

Deafness and Tinnitus

Any relative with NF-2

Xeroderma pigmentosum ( XP) (Choice E) is an autosomal recessive genetic disorder of DNA repair in which the ability to repair damage caused by
ultraviolet (UV) light is deficient.Multiple basal cell carcinomas (basaliomas) and other skin malignancies frequently occur at a young age in those with
XP. In fact, metastatic malignant melanoma and squamous cell carcinoma are the two most common causes of death in XP. XP is roughly six times
more common in Japanese people than in other groups. Some of the most common symptoms of XP include:

A severe sunburn when exposed to only small amounts of sunlight. Often occurring during a child's first exposure to sunlight.

Development of many freckles at an early age

Rough-surfaced growths (solar keratoses), and skin cancers

Eyes that are painfully sensitive to the sun and may easily become irritated, bloodshot, and clouded

Blistering or freckling on minimum sun exposure

Spidery blood vessels

Oozing raw skin surface

Limited growth of hair on chest and legs

Scaly skin

Irregular dark spots on the skin

You are asked to see a 5 year old boy brought into the ER by his parents after the develpment of a blistering rash. Physical exam reveals vesicles and
bullae on the lips, mouth, and conjunctivae, along with ulcerations and crusting, denuded skin. Vitals signs are within normal limits. A detailed history
reveals only that he has just completed 10 days of therapy with trimethoprim-sulfamethoxazole for a urinary tract infection.
Which of the following bacteria has been most credibly associated with the pathogenesis of this disease?

1)
2)
3)
4)
5)

Streptococcus pneumoniae
Mycoplasma pneumoniae
Staphylococcus aureus
Haemophilus influenzae
Staphylococcus epidermidis

The correct answer is choice B.

This vignette describes an antibiotic-associated case of Stevens-Johnson syndrome (SJS). SJS is thought to be an immune complex-mediated
hypersensitivity reaction. It is characterized by large vesicles and bullae that rupture, leaving raw, denuded areas that are predisposed to infection.
The most common areas affected are the face, trunk, and extremities. For diagnosis, the rash must involve two or more mucosal surfaces, including the
eyes, oropharynx, anogenital region, and viscera of the gastrointestinal and/or respiratory tract.

The causes of SJS are numerous, some with more evidence behind their causation than others. Causes fall under the four categories of infectious,
drug-related, malignancy, and idiopathic. Various causes of the first three categories are as follows:

1. Infectious: Mycoplasma pneumoniae, group A streptococci, influenza virus, herpes simplex virus, AIDS, coccidioidomycosis, and malaria

2. Drug-related: sulfa drugs, penicillins, and anticonvulsants

3. Malignancy: carcinomas and lymphomas

The clinical course of SJS can take up to six weeks to resolve, often leaving patients with impaired vision, strictures in the gastrointestinal,
genitourinary, and respiratory tracts, and/or diffuse scarring. Treatment focuses on supportive care, including immediate withdrawal of the potential
offending agent, hydration, keeping wounds sterile, pain control, antibiotics if infection is suspected, and close monitoring for development of
complications, usually in the setting of an intensive care unit. Corticosteroids and intravenous immunoglobulin have also been used, but their

effectiveness needs further investigation.

SJS must be differentiated from toxic epidermal necrolysis, which is thought to be a very severe form of SJS requiring even more intensive, focused
treatment efforts.

Other bacteria (choices A, C, D, and E) may have a role in the pathogenesis of SJS, but none of these is as well-studied and linked to the disease as
is Mycoplasma pneumoniae.

A 2 month old baby presents to your office with a yellowish, greasy, scaling area of skin on the scalp. On physical exam, you note that it is
also present on the forehead, eyebrow area, and behind the ears. There is no associated erythema or discharge, nor is it present anywhere else but
the head. The baby is afebrile and otherwise healthy.
The most appropriate next step in management is which of the following?
1)
2)
3)
4)
5)

1% hydrocortisone cream
frequent shampooing and brushing of the region
baby oil application
tar-based shampoo
antifungal shampoo

The correct answer is B.

This baby has seborrheic dermatitis, commonly known as "cradle cap" when present in infants. Seborrheic dermatitis is a common condition and affects
infants, older children, and adults. In infants, it is thought to be caused by the passage of maternal hormones to the newborn. There is also some
evidence that the presence of Malassezia furfur, a fungus, contributes to the condition since antifungal preparations have been shown to be effective in
treatment.

Cradle cap is characterized by yellowish or mildly erythematous areas of greasy, scaling skin on the scalp, forehead, eyebrow region, ear, and diaper
region. In its diffuse form, it can be present on the entire body. It does not appear to be distressing to the infant nor does it imply any mistreatment or
poor hygiene. Most often, the condition clears on its own without any intervention. However, when supportive care fails to result in improvement,

there are a number of options for treatment. The primary intervention recommended is frequent shampooing of the affected regions to loosen the
scales followed by brushing out of the scales. In most situations, this is all that is required for improvement.

If shampooing and brushing fails, baby oil application (choice C) followed by brushing out of the scales is commonly suggested. Baby oil application,
however, can be tedious and often does not yield better results than shampooing alone.

Medicated shampoos such as those that contain tar (choice D), selenium sulfide, salicylic acid, or antifungals (choice E; typically, ketoconazole) are
other options for treatment if the cradle cap is severe or fails to respond to other milder solutions. These medicated shampoos are the most common
treatment for older children or adults with seborrheic dermatitis.

One percent hydrocortisone cream (choice A) is another effective agent that primarily works to treat the secondary irritation caused by the cradle cap.
Steroids should be used sparingly in infants and discontinued once the desired effect is achieved.

An 18 year-old male college student who admits to a recent episode of unprotected intercourse presents with burning, stinging lesions that developed
on the shaft of his penis 5 days ago. His physical examination is consistent with genital herpes and you advise him of your presumptive diagnosis. You
inform him that in order to ensure diagnostic accuracy you need to culture the lesions, but that you feel he should started on an antiviral regimen to aid
in both the reduction and duration of symptoms of what appears to be an initial outbreak of genital herpes.
Antiviral regimens that are appropriate for the treatment of an initial episode of genital herpes include each of the following EXCEPT:
1)
2)
3)
4)
5)

Acyclovir 200 mg orally 5 times daily for 7 to 10 days


Acyclovir 400 mg orally 3 times daily for 7 to 10 days
Famciclovir 250 mg orally 3 times daily for 7 to 10 days
Ganciclovir 1000 mg orally 3 times daily for 7 to 10 days
Valacyclovir 1 g orally twice daily for 7 to 10 days

The correct answer is choice D

Ganciclovir is a synthetic guanine derivative that is effective against cytomegalovirus but has no FDA indication for and thus is not used in, the
management of initial episodes of genital herpes. It is also indicated for the treatment of CMV retinitis in immunocompromised patients, including

patients with acquired immunodeficiency syndrome (AIDS). Ganciclovir brand name products include CYTOVENE-IV, which is also indicated for the
prevention of CMV disease in transplant recipients at risk for CMV disease; and ZIRGAN: a topical ophthalmic antiviral that is indicated for the treatment
of acute herpetic keratitis (dendritic ulcers).

Acyclovir (Zovirax) 200 mg orally 5 times daily for 7 to 10 days (Choice A) and Acyclovir (Zovirax) 400 mg orally 3 times daily for 7 to 10 days (Choice
B) are both correct answers because they are both FDA approved dosage regimens for an initial outbreak of genital herpes.

Famciclovir (Famvir) 250 mg orally 3 times daily for 7 to 10 days (Choice C) is correct because this dosing regimen is FDA-approved for the
management of an initial episodes of genital herpes. Although efficacy data between acyclovir, famciclovir, and valacyclovir is similar, Famvir and Valtrex
receive the majority of prescriptions for this condition.

Valacyclovir (Valtrex) 1 g orally twice daily for 7 to 10 days (Choice E) is correct because this dosing regimen is approved for the management of an
initial genital herpes episode.

Figure 1: Acute management of HSV review

A 65-year-old woman is evaluated for an ulcer on her lower leg that has been present for one month. Her past medical history includes hypertension
and mild heart failure. Her current medications are furosemide and enalapril. Examination shows lower leg edema with hyperpigmentation and a
shallow, 3 cm ulcer located 4 cm above the right medial malleolus. The base of the ulcer is pink. The ankle-brachial index is normal. Fasting blood

glucose is 110 mg/dL (6.1 mmol/L).


Which of the following is the most likely diagnosis?
1)
2)
3)
4)
5)

Diabetic ulcer
Lymphedema
Pressure ulcer
Arterial insufficiency
Venous insufficiency

The correct answer is Choice E.

The classical description of a venous ulcer consists of coexisting varicosities with the ulcer located in the gaiter area; shallow, painful; granulation tissue
and fibrin present. The associated findings include edema, venous dermatitis, and lipodermatosclerosis.

Venous ulcers have a high prevalence and are more common in women than in men. The most frequent causes of lower extremity ulcers are venous
insufficiency (Choice E), arterial insufficiency, neuropathy (often related to diabetes), and ulcers from prolonged pressure and ischemia.

Reduction in the severity and frequency of venous leg ulcers can be achieved by managing peripheral oedema over the long term. Patient education and
prevention of pressure-related injury can reduce skin integrity problems in the presence of arterial disease. Successful management of leg ulcers
requires a clear diagnosis, establishment of a treatment plan, accurate monitoring, and adherence to the plan as the ulcer decreases in size. Once
healed, long-term maintenance compression is crucial.

Neuropathic ulcers (Choice A), including those secondary to diabetic neuropathy, usually occur in the plantar aspect of feet. Note that this type of ulcer
may occur with neuropathies secondary to diseases other than diabetes mellitus.

Pressure ulcers (Choice C) are usually located over bony prominences in patients with limited mobility. The risk factors are excessive moisture and
altered mental status.

Arterial insufficiency (Choice D) associated leg ulcers are commonly deep, located over bony prominences; round or punched out with sharply
demarcated borders, yellow base, or necrosis; exposure of tendons. Associated findings include abnormal pedal pulses, cool limbs, femoral bruit, a low
ankle-brachial and prolonged venous filling time.

A 6 year old child presents with flesh color papules on the hand that are not pruritic. Examination reveals lesion that are approximately 4 mm in
diameter with central umbilication. A halo is seen around those lesions undergoing regression.
Which of the following is the most likely diagnosis?

1)
2)
3)
4)
5)

verruca vulgaris
mollucum contagiosum
keratoacanthoma
herpetic whitlow
hemangioma

The correct answer is Choice B.

Molluscum contagiosum (MC) is a self-limited epidermal viral (Pox virus) infection, characterized clinically by skin-colored papules that are often

umbilicated, occurring in children and sexually active adults. Individual lesions last weeks to month and resolve spontaneously. In immunocompetent
children and sexually active adults painful aggressive therapy is not indicated. In HIV-infected individuals, however, numerous large mollusca often arise
on the face, causing significant cosmetic disfigurement. Prevalence of molluscum contagiosum in HIV positive patients may be as high as 5-18 %.
Transmission has been reported by direct skin contact and has occured in wrestlers, children sharing baths, towels, gymnasium equipment, and
benches. Treatment modalities comprises awaiting natural involution, destructive options such as curretage, cryotherapy, topical vesicants
(Cantharone/Cantharone Plus and Imiquimod.The awaiting natural involution is the most common approach and is the best management option.

Warts (Verruca Vulgaris) (Choice A) are benign proliferations of skin and mucosa caused by the human papillomavirus (HPV). Currently, more than 100
types of HPV have been identified. Certain HPV types tend to occur at particular anatomic sites; however, warts of any HPV type may occur at any site.
The primary clinical manifestations of HPV infection include common warts, genital warts, flat warts, and deep palmoplantar warts (myrmecia).Common
warts also are termed verruca vulgaris. They appear as hyperkeratotic papules with a rough, irregular surface. They range from smaller than 1 mm to
larger than 1 cm. They can occur on any part of the body but are seen most commonly on the hands and knees A claster of skin-colored, umbilicated
papules is not consistent with warts (Choice A).

Keratoacanthoma (KA) (Choice C) is a relatively common low-grade malignancy that originates in the pilosebaceous glands and closely and
pathologically resembles squamous cell carcinoma (SCC). In fact, strong arguments support classifying keratoacanthoma as a variant of invasive SCC.
Lesions typically are solitary and begin as firm, roundish, skin-colored or reddish papules that rapidly progress to dome-shaped nodules with a smooth
shiny surface and a central crateriform ulceration or keratin plug that may project like a horn. Most keratoacanthomas occur on sun-exposed areas. The
face, neck, and dorsum of the upper extremities are common sites. Truncal lesions are rare. Lesions usually are skin-colored to pinkish-red. Unaffected
skin retains its normal appearance. Didderent clinical presentation omits KA.

Herpetic whitlow (Choice D) is an intensely painful infection of the hand involving 1 or more fingers that typically affects the terminal phalanx.1 Herpes
simplex virus 1 (HSV-1) is the cause in approximately 60% of cases of herpetic whitlow, and herpes simplex virus 2 (HSV-2) is the cause in the
remaining 40%.Involved finger is often exquisitely tender and quite edematous; however, in contrast to a felon, the pulp space usually is not tensely
swollen. Examination usually reveals the characteristic grouped vesicular lesions or ulcers with surrounding erythema. Fluid within the vesicles is usually
clear, although it may appear cloudy or hemorrhagic.

Hemangioma (Choice E) is the most common tumor of infancy. Soft, bright red to deep purple, compressible. On diascopy, does not blanch completely
and on exam, clinically presents as a nodule or plaque, 1 to 8 cm in size.The incidence in newborns is between 1 and 2.5%; in white children by 1 year
of age it is 10%. Females more affected than males by a 3 to 1 ratio. HOI is a localized proliferative process of angioblastic mesenchyme. The initial
proliferative phase lasts from 3 to 9 months, sometimes more. HOI usually enlarge rapidly during the first year. In a subsequent phase of involution

about the 95% of HOI regress, and this occurs gradually over 2 to 6 years and is usually complete by the age of 10. Involution varies greatly between
individuals and is not correlated with size, location, or appearance of the lesion.

You are a senior resident in pediatrics and get called by the nurse to evaluate the rash of a newborn who is 5-day-old. On examination there are
indurated nodules and plaques on the arms and buttocks. The overlying skin is purple. Despite skin changes the child is doing well and vitals are within
normal limits. The birth history is unremarkable except meconium aspiaration. You diagnosed the Subcutaneous fat necrosis of the newborn.
The most common finding associated with subcutaneous fat necrosis of the newborn includes which of the following:

1)
2)
3)
4)
5)

Hypercalcemia
liver necrosis
type I diabetes
cellulitis
ulceration

The correct answer is Choice A


Subcutaneous fat necrosis of the newborn (SCFN) is an uncommon disorder. Newborns who develop SCFN usually are healthy and full-term at delivery
but have had some antecedent obstetric trauma, meconium aspiration, asphyxia, hypothermia, or peripheral hypoxemia. Within the first several days to
weeks of life, hard, indurated nodules and plaques with ill-defined overlying erythema develop on the trunk, arms, buttocks, thighs, or cheeks. The

lesions are not warm. Pain may occur, with a frequency as high as 25% in one series. The infants usually appear well and are afebrile. The condition
begins as an area of edema and progresses to variably circumscribed nodules and plaques that have a deep, indurated feel, implying a panniculitis. The
overlying skin may be red, purple, or flesh-colored and may look taut and shiny.
The exact pathogenesis of SCFN in not known. It is postulated that cold or stress-induced injury to immature fat results in the development of
solidification and necrosis. A granulomatous infiltrate forms subsequently and nonrenal absorption of calcium increases. Staining of biopsy specimens
demonstrates increased levels of 25-hydroxyvitamin D3 -1 alpha-hydroxylase within the granulomatous infiltrate as is seen in other granulomatous
conditions such as sarcoidosis. No other organ systems are involved, unlesshypercalcemia intervenes. SCFN of the newborn is a harmless, self-limited
condition. Significant morbidity (seizures, blindness, failure to thrive) and even mortality (from infection and cardiac arrest) can result from the
associated hypercalcemia.
Liver necrosis (Choice B), type 1 Diabetes Mellitus (Choice C), cellulitis (Choice D) and ulceration (Choice E) are not commonly associated with SCFN.

A previously healthy 10-year-old girl presented with a 3-week history of a rash confined to the extremities, which waxes and wanes, but does not
disappear and worsens after physical activity. She did complain of fevers that have subsided as well as a sore throat and some mild muscle aches. She
is otherwise healthy and is on no medications. She had a febrile seizure when she was 2 years of age but has no other significant past medical history.
The most likely diagnosis is which of the following:

1)
2)
3)
4)
5)

Roseola
childhood sarcoidosis
erythema infectiosum
rheumatic fever
rubella

The correct answer is Choice C

Parvovirus B19 infection (Erythema Infectiousum) is common in school-aged and younger children who attend daycare facilities. In general, children
transmit the virus to parents and siblings. In young children, the antibody seroprevalence ranges from 5-10%. This increases to 50% in adolescents and
approaches 90% in the elderly. Common symptoms of parvovirus B19 (B19V) infection include a mild nonspecific prodromal illness that may consist of
fever (15-30% of patients), malaise, headache, myalgia, nausea, and rhinorrhea; typically beginning 5-7 days after initial infection. These symptoms
correspond to the initial viremia and dissipate in 2-3 days.

Approximately 1 week later, a bright red macular exanthem appears on the cheeks and is often associated with circumoral pallor. A diffuse
maculopapular rash (erythematous macules and papules that become confluent, giving a lacy or reticulated appearance) can appear 1-4 days later and
fades to a lacy erythematous rash, which may be pruritic and may spread gradually toward the distal extremities. The rash can waxe and wane, but
does not disappear and often worsens after physical activity (Choice C). Most seropositive patients have no history of this classic biphasic illness. The
clinical symptoms widely vary, and the classic "slapped cheek" rash is much more common in young children. Often, Parvovirus B19 can cause selflimited arthralgias, lasting 3 weeks; but may persist for several months or years.

Roseola infantum (Choice A) is a common childhood disease that is caused by human herpesvirus 6 (HHV-6). The classic presentation of roseola
infantum is a 9- to 12-month-old infant who acutely develops a high fever and often a febrile seizure. After 3 days, a rapid subsidence of fever to
normal temperature occurs, and a morbilliform rash appears. Similarly to other herpes viruses, HHV-6 remains latent in most patients who are
immunocompetent. HHV-6 is a major cause of morbidity and mortality in patients who are immunosuppressed. In the primary infection, replication of
the virus occurs mainly in the leukocytes and the salivary glands. Serologic tests indicate that human herpesvirus 6 (HHV-6) infection is common among
the population. In roseola infantum, age ranges from 2 weeks to 3 years.The HHV-6 has been reported to be responsible for 10-45% of cases of febrile
illness in infants. The population-based study revealed primary HHV-6 infection cumulative percentages of 40% by age 12 months and 77% by age 24
months.The peak age of acquisition of primary HHV-6 infection is 9-21 months. Despite the high fever, few clinical findings are observed early in the
course of roseola infantum. The lack of upper respiratory tract infection is notable, and meningeal signs and encephalopathy are not present.
Gastrointestinal symptoms, signs of electrolyte imbalance, or evidence of dehydration are rarely present. A febrile seizure, with no residual findings, may
have occurred. After an abrupt loss of fever, the characteristic rash appears. The eruption is generalized and composed of discrete, small, pale pink
papules or a blanchable, maculopapular exanthem that is 1-5 mm in diameter. This rash may last 2 days. The characteristic enanthem (Nagayama
spots) consists of erythematous papules on the mucosa of the soft palate and the base of the uvula. The enanthem may be present on the fourth day in
two thirds of patients with roseola.

Childhood Sarcoidosis (Choice B) is a systemic granulomatous disease of unknown etiology that most commonly affects young adults, who frequently

present with hilar lymphadenopathy, pulmonary infiltration, and ocular and cutaneous lesions. Sarcoidosis is a chronic inflammatory disease
characterized by a highly focused exaggerated immune response to an unknown antigen at the target organs. Two distinct forms of childhood
sarcoidosis are noted. Older children usually present with a multisystem disease similar to the adult manifestation with frequent lymphadenopathy and
pulmonary involvement, as well as generalized signs and symptoms, such as fever and malaise. In contrast, early onset childhood sarcoidosis is a
unique form of the disease characterized by the triad of rash, uveitis, and arthritis in presenting patients who are younger than 4 years. Cutaneous
involvement occurs in about 24-40% of older children with sarcoidosis and in 77% of young children with sarcoidosis. The most frequent cutaneous
eruptions include soft, redtoyellowish brown, or violaceous flat-topped papules, most frequently found on the face. Larger, violaceous, plaquelike
lesions may be found on the trunk, extremities, and buttocks. Other skin lesions of sarcoidosis include nodules, hyperpigmented lesions, hypopigmented
lesions, ulcers, subcutaneous tumors, and erythema nodosum.

Rheumatic fever (RF) (Choice D) is a systemic illness that may occur following group A beta hemolytic streptococcal (GABHS) pharyngitis in children.
Acute rheumatic fever (RF) is a systemic disease. Thus, patients may present with a large variety of symptoms and complaints. The RF requires the
presence of 2 major or 1 major and 2 minor criteria for the diagnosis of rheumatic fever. Having evidence of previous group A streptococci (GAS)
pharyngitis is also necessary. These criteria are not absolute, and the diagnosis of rheumatic fever can be made in patients with only confirmed
streptococcal pharyngitis and chorea.

Major diagnostic criteria

Carditis

Polyarthritis

Chorea

Subcutaneous nodules

Erythema marginatum

Minor diagnostic criteria

Fever

Arthralgia

Prolonged PR interval on electrocardiography

Elevated acute-phase reactants (APRs) (erythrocyte sedimentation rate and C-reactive protein)

The skin involvement comprises Erythema Marginatum (erythema annulare) that occurs in 5-13% of patients with acute rheumatic fever. EM begins as
1-cm to 3-cm diameter, pink-to-red nonpruritic macules or papules located on the trunk and proximal limbs but never on the face. The lesions spread
outward to form a serpiginous ring with erythematous raised margins and central clearing. The rash may fade and reappear within hours and is
exacerbated by heat. Thus, if the lesions are not observed easily, they can be accentuated by the application of warm towels, a hot bath, or the use of
tangential lighting. The rash occurs early in the course of the disease and remains long past the resolution of other symptoms.

Rubella (Choice E) is usually a mild viral (RNA virus classified as a Rubivirus in the Togaviridae family) illness involving the skin, the lymph nodes, and,
less commonly, the joints.In children, a prodrome may not be present. The rash may be the first manifestation. In adults, fever, sore throat, and rhinitis
may be present. The exanthem begins as discrete macules on the face that spread to the neck, the trunk, and the extremities. The macules may
coalesce on the trunk. Appearance of the rash corresponds with the appearance of the rubella-specific antibody. The exanthem lasts 1-3 days, first

leaving the face, and may be followed by desquamation. On occasion, a nonspecific enanthem (Forchheimer spots) of pinpoint red macules and
petechiae can be seen over the soft palate and the uvula just before or with the exanthem.

A 57 year-old man presented to your office with multiple skin lesions of the eyebrow, flank and mid-back. At this time, the clinical differential diagnosis
included neurofibromata, dysplastic nevi, and malignant melanoma. His previous surgical history was significant for prior removal of two atrial polyps in
1996 and 2003. The lesion from his skin showed lentigenous melanocytic hyperplasia when it was biopsied. The histology of the atrial lesions showed a
myxomatous polyp. The patient was eventually diagnosed with a Carney complex syndrome.
Which of the following conditions is not part of the diagnostic criteria of Carney complex?
1)
2)
3)
4)
5)

Acromegaly, ovarian cysts


Cardiac myxomas, primary pigmented nodular adrenocortical disease (PPNAD)
Hashimotos thyroiditis, mesothelioma
Psammomatous melanotic schwannoma, mucosal myxoma
First degree relative affected with Carney complex, blue nevus

The correct answer is choice C.

Carney complex (CNC) is a familial multiple neoplasia characterized by cardiac and cutaneous myxomas, multiple endocrine tumors (in the pituitary,
thyroid, ovaries and testicles), primary pigmented nodular adrenocortical disease (PPNAD), breast and peripheral nervous system tumors and cutaneous
lentiginosis syndrome. Generally disorders involving autoimmunity such as Hashimotos thyroiditis or lung cancers like mesothelioma are not caused by
endocrine disturbances and hence not included in the carney complex.

The Carney complex is an autosomally dominant inherited syndrome. The clinical manifestations are very variable among patients, even in the same
family. Approximately half of the families with CNC that have been studied presented mutations in the gene PRKAR1A, which is located on the long arm
of chromosome 17 (17q22-24). This gene acts as a classical tumor suppressor, responsible for the production of the type 1 regulatory subunit of protein
kinase (PKA). PKA is related to important pathways for endocrinal signaling. The R1 subunit inhibits PKA function, and PRKAR1A mutations originate a
truncated protein that is functionally null, thus leading to increased intracellular signaling via PKA, and consequent endocrinal hyperactivity or tumor
formation.

In order to be diagnosed as a case of Carney complex, a patient must either:

1) exhibit two of the manifestations of the diseases listed, or

2) exhibit one of these manifestations and meet one of the supplemental criteria.

1. Spotty skin pigmentation with typical distribution (lips, conjunctiva and inner or outer canthi, vaginal and penile mucosa)

2. Myxoma (cutaneous and mucosal)

3. Cardiac myxoma

4. Breast myxomatosis or fat-suppressed magnetic resonance imaging findings suggestive of this diagnosis

5. Primary pigmented nodular adrenal disease or paradoxical positive response of urinary glucocorticosteroids to dexamethasone administration
during Liddle test

6. Acromegaly due to growth hormone-producing adenoma

7. Large cell calcifying Sertoli cell tumor or characteristic calcification on testicular ultrasonography

8. Thyroid carcinoma or multiple, hypoechoic nodules on thyroid ultrasonography, in a young patient

9. Psammomatous melanotic schwannoma

10. Blue nevus, epithelioid blue nevus (multiple)

11. Breast ductal adenoma (multiple)

12. Osteochondromyxoma

Supplemental criteria:

1. Affected first-degree relative

2. Inactivating mutation of the PRKAR1A gene

A six-month-old infant with generalized seborrheic dermatitis presents with frequent loose bowels and weight loss inconsistent with previous growth
trends. His weight is currently below the 3rd percentile and has not gained any weight in the past 2 months. His birth history was significant for an
emergency c-section as a result of decelerations but apgar scores were normal at birth.

Which one of the following would be the most appropriate next step?

1)
2)
3)
4)
5)

Reassure the parents that the symptoms are indicative of a self-limited process
Perform a work-up to detect immunodeficiencies
Prescribe oral course of steroids and schedule barium enema
Apply a selenium- zinc shampoo twice weekly
None of the above as the disease will resolve on its own

The correct answer is Choice B

Seborrheic dermatitis (SD) is a papulosquamous disorder patterned on the sebum-rich areas of the scalp, face, and trunk. In addition to sebum, this
dermatitis is linked to Malassezia, immunologic abnormalities, and activation of complement. It is commonly aggravated by changes in humidity,
changes in seasons, trauma (eg, scratching), or emotional stress. The severity varies from mild dandruff to exfoliative erythroderma. Seborrheic
dermatitis is associated with normal levels of Malassezia but an abnormal immune response. Helper T cells and antibody titers are depressed compared
with those of control subjects. The contribution of Malassezia species to seborrheic dermatitis may come from its lipase activityreleasing inflammatory
free fatty acidsand from its ability to activate the alternative complement pathway.

Malassezia organisms are probably not the cause but are a cofactor linked to a T-cell depression, increased sebum levels, and an activation of the

alternative complement pathway. Persons prone to this dermatitis also may have a skin-barrier dysfunction. The prevalence rate of seborrheic dermatitis
is 3-5%, with a worldwide distribution. Dandruff, the mildest form of this dermatitis, is probably far more common and is present in an estimated 1520% of the population. The usual onset occurs with puberty. It peaks at age 40 years and is less severe, but present, among older people. In infants, it
occurs as cradle cap or, uncommonly, as a flexural eruption or erythroderma. Generalized seborrheic erythroderma is rare. It occurs more often in
association with AIDS, congestive heart failure, Parkinson disease, and immunosuppression in premature infants. In the case the child has a generalized
skin process , changes in weight and bowel habits that can be a sign of some systemic disease with immunodeficiency or immunosuppression . That is
why reassurance of the parents that the symptoms are indicative of a self-limited process (Choice A) is not a correct answer. It is essential to find a
cause of the immunodeficiency(Choice B).

Topical corticosteroids and especially oral corticosteroids may hasten recurrences, may foster dependence because of a rebound effect, and are
discouraged except for short-term use. So, oral corticosteroids plus barium enema (Choice C) is a wrong answer.

Skin involvement responds to ketoconazole, naftifine, or ciclopirox creams and gels. Alternatives include calcineurin inhibitors (ie, pimecrolimus,
tacrolimus), sulfur or sulfonamide combinations, or propylene glycol and a selenium- zinc shampoo twice weekly (Choice D). As immunodeficiency is the
likely cause, topic treatment will not likely resolve the dermatitis. Class IV or lower corticosteroid creams, lotions, or solutions can be used for acute
flares. Systemic ketoconazole or fluconazole may help if seborrheic dermatitis is severe or unresponsive. Combination therapy has been recommended.

Also, in patients with prominent immunosuppression, SD will not resolve on its own (Choice E).

A 4 year old boy with known history of atopic dermatitis presents to your office with an excoriated, eczematous, scaling, intensely itchy rash on his
cheeks, arms, and trunk. His atopic dermatitis appears to be poorly controlled despite judicious application of moisturizer and petroleum preparations
by his parents. Vital signs are within normal limits, and the rest of his physical exam appears benign.
What is the most appropriate next step in the management of this patient?
1)
2)
3)
4)
5)

a course of oral cephalexin (Keflex)


antihistamines
reassurance and close observation
topical corticosteroids
a course of oral prednisolone

The correct choice is D.

Atopic dermatitis (AD; also referred to as eczema) is a common dermatological disease that commonly appears in the first year of life, with most
patients being formally diagnosed by 5 years of age. It is commonly associated with future development of asthma and allergic rhinitis in the so-called
"atopic triad" or "atopic march." AD is characterized by scaling, erythematous papules that are often intensely pruritic. In the first year of life, it
commonly appears on the cheeks and extensor surfaces and progresses to the flexor surfaces in childhood. It is important to note that eczema
generally spares the diaper area.

First-line treatment involves judicious use of moisturizers and ointments, lukewarm baths, and sometimes wet, occlusive dressings to the affected
areas. If conservative treatment fails, the mainstay of medical treatment is daily topical corticosteroids, usually in the mid-potency range.

Oral steroids (choice E) are not usually indicated in the treatment of AD. In addition to the rebound flare of AD that is common after steroid use,
systemic corticosteroids are best avoided in children if topical preparations have not yet been exhausted.

Reassurance and close observation (choice C) is not indicated in this child. Even if the presentation were a very mild, first-time rash, moisturizer
treatment would be recommended.

Oral antibiotics with good Streptococcal and Staphylococcal coverage, such as cephalexin (Keflex; choice A) are indicated if the AD appears infected.
Super-infected AD can present as an increasingly erythematous, weeping rash with or without fever. This vignette does not describe an infected case of
AD.

Antihistamines (choice B) may alleviate some of the pruritis associated with AD and are useful as an adjunct to topical steroids but will not help heal the
condition alone.

A 5-year-old child is febrile for 5 days and has bilateral conjunctival injection, cervical lymphadenopathy, dry, cracked, cherry red lips, swelling of the

hands and feet, and truncal polymorphous rash. On examination the child has a temperature of 40 C (104 F), heart rate of 125/min, respirarory rate
of 26/min and a blood pressure of 90/60 mm Hg. The child is fairly irritable but is alert.
Which of the following is the most appropriate treatment?

1)
2)
3)
4)
5)

Aspirin plus prednisone


Aspirin plus IVIG (gamma globulin)
Dicloxacillin
Acetaminophen plus IVIG (gamma globulin)
Prednisone plus IVIG (gamma globulin)

The correct answer is Choice B

Kawasaki disease (mucocutaneous lymph node syndrome) is a multisystemic acute large- and medium-vessel vasculitis that primarily affects young
children (80% are < 5 years of age) with predilection for those of Asian ancestry. Epidemics occur primarily in late winter and spring with 3-year
intervals, lending some credence to the possibility of an infectious etiology. Kawasaki disease is most commonly observed in children from the middle
and upper-middle classes.

The estimated number of children hospitalized annually in the United States is about 3000, though more than 4000 admissions occurred in 2006, some
of which were incomplete cases. The highest incidence in the United States is in Hawaii at 17 per 100,000 children younger than 5 years. Outside the
United States, the disease is most frequently observed in Japan, Taiwan, and Korea. The prevalence of Kawasaki disease increased from 1967 to the
mid 1980s and has leveled out at 5000-6000 cases per year. This condition is divided into acute,subacute, and chronic phases. Diagnostic criteria

include the following:

Lasts 12 weeks and presents with the following symptoms:

Fever persiting (usually > 40C (104 F) for at least five days AND 4 of the criteria below:

Bilateral, nonexudative, painless conjunctivitis.

Polymorphous rash (primarily truncal).

Cervical lymphadenopathy (often unilateral, with at least one node >1.5 cm).

Diffuse mucous membrane erythema (e.g., strawberry tongue, red fissured lips).

Erythema of the palms and soles; indurative edema of the hands and feet; late desquamation of the fingertips (in subacute phase).

Kawasaki disease can cause several serious complications that determine the degree of disability. One of the most dangerous complications of the
Kawasaki disease is the development of coronary artery aneurysms (CAA) with consequences such as thrombosis or rupture. Acute myocardial
infacrction (MI) has been reported secondary to true coronary artery obstruction. These aneurysms tend to develop approximately 1-2 weeks after
onset of symptoms. Myocarditis, congestive heart failure, pericarditis with pericardial effusion, mitral or aortic insufficiency, and dysrhythmias may be
observed early in the disease. Decreased left ventricular function is present in approximately 50% of all patients with Kawasaki syndrome. Additionally,

arthritis persists in some children.

The pathophysiology of Kawasaki disease involves inflammation. The patient's own immune system probably causes the vasculitis that leads to
morbidity and mortality in Kawasaki disease. Early and aggressive intervention improves outcome. Standard treatment includes aspirin and intravenous
immunoglobulin (IVIG) (Choice B) to treat inflammation and to prevent consequences of coronary artery disease. IVIG neutralizes circulating myelin
antibodies by means of anti-idiotypic antibodies; downregulates proinflammatory cytokines, including INF-gamma; blocks Fc receptors on macrophages;
suppresses inducer T and B cells and augments suppressor T cells; blocks complement cascade; promotes remyelination. Aspirin inhibits prostaglandin
synthesis, which prevents formation of platelet-aggregating thromboxane A2. Adequate anti-inflammatory therapy requires that aspirin be combined
with gamma globulin.

Aspirin plus prednisone (Choice A), Dicloxacillin (Choice C), Acetaminophen plus gamma globulin (Choice D) and Prednisone plus gamma globulin
(Choice E) are wrong answers.

A 21-year-old man presents at your clinic with recurrent episodes of flexural eczema, contact urticaria, and recurrent skin infections. He also complains
of severe abdominal cramps and diarrhea after eating seafood the previous evening. He has no pre-existing medical conditions and is on no current
medications.
Which condition is he most likely suffering from?
1)
2)
3)
4)
5)

Seborrheic dermatitis
Atopic dermatitis
Airborne contact dermatitis
Nummular dermatitis
Stasis dermatitis

The correct answer is Choice B.

The patient has atopic dermatitis (Choice B) secondary to seafood allergy. Recurrent bacterial, fungal, and viral skin infections are a frequent
complication of atopic dermatitis. It is believed that the lesions of atopic dermatitis provide an environment that allows bacteria, fungi, and viruses to
easily invade the skin and replicate. People with atopic dermatitis are predisposed to contact urticaria. Flexural eczema is also a common symptom.

Atopic dermatitis usually presents during early infancy and childhood, but it can start in adulthood. The lifetime prevalence of atopic dermatitis is 10
20% in children and 13% in adults. Its prevalence has increased 2-3-fold during the past 30 years in industrialized countries. The primary treatment
involves prevention, and includes avoiding or minimizing contact with known allergens. Once that has been established, topical treatments can be used.
Topical treatments focus on reducing both the dryness and inflammation of the skin.

Seborrhoic dermatitis (Choice A) has certain features that resemble atopic dermatitis but it usually starts before the age of 2 months and clears
spontaneously within 3 or 4 weeks. The essential features are erythema and scaling. Unlike atopic dermatitis, the scalp is always involved.

Airborne contact dermatitis (Choice C) is caused by an allergen or an irritating substance in the air. Irritant contact dermatitis accounts for 80% of all
cases of contact dermatitis. This dermatitis usually occurs maximally on the eyelids, but it may affect other areas exposed to chemicals in the air,
particularly the head and the neck.

Nummular dermatitis (Choice D) is a less common type of dermatitis, with no known cause and which tends to appear more frequently in middle-aged
people.

Stasis dermatitis (Choice E) is an inflammation on the lower legs which is caused by build-up of blood and fluid and it is more likely to occur in people
with varicose.

A woman presents to a dermatologist because she has lost almost all the hair on her body, including scalp hair, eye brows, eye lashes, arm pit and
groin hair, and the fine hairs on her body and extremities.
She most likely has a variant of which of the following?

1)
2)
3)
4)
5)

Alopecia areata
Androgenic alopecia
Cutaneous lupus erythematosus
Lichen planus
Trichotillomania

The correct answer is choice A.

This is a case of alopecia areata, universal pattern. Alopecia areata is a recurrent nonscarring type of hair loss that can affect any hair-bearing area. The
pathophysiology of alopecia areata remains unknown. The most widely accepted hypothesis is that alopecia areata is a T-cellmediated autoimmune
condition that is most likely to occur in genetically predisposed individuals.

Alopecia areata can be classified according to its pattern into:

localized pattern occurs when there is only one or few scattered patches.

Reticular pattern occurs when hair loss is more extensive and the patches coalesce.

Ophiasis pattern occurs when the hair loss is localized to the sides and lower back of the scalp

Sisaipho (ophiasis spelled backwards) pattern occurs when hair loss spares the sides and back of the head

Alopecia totalis occurs with 100% hair loss on the scalp

Alopecia universalis occurs with complete loss of hair on all hair-bearing areas

Most patients have only a few focal areas of alopecia, and spontaneous regrowth usually occurs within 1 year. Extensive forms of alopecia areata only
forms 10% of cases and less than 1% have alopecia universalis. Patients with extensive longstanding conditions are less likely to experience significant
long-lasting regrowth.

This image shows a treatment algorithm for alopecia areata. However, it should be noted that treatment is not mandatory because the condition is
benign, and spontaneous remissions and recurrences are common.

Androgenic alopecia (choice B) mainly affects men and when affecting women it is only presented by gradual diffuse thinning of hair on the crown.

Cutaneous lupus erythematosus (choice C) and lichen planus (choice D) may cause localized hair loss only if involving a hairy area.

Trichotillomania (Choice E) is a self-induced primary psychiatric disorders resulting from repetitive hair manipulations by the patient's own hand.

During the examination of the the 2-month-old female you note a 2-cm bright red nodular plaque on her right thigh. Besides this nodular plaque there
are no other findings.The child is doing well and vitals are within normal limits. Development history is normal and immunization is up to date.
Which of the following is the correct statement regarding hemangiomas?

1)
2)
3)
4)
5)

they usually will require surgery


most present in the teenage years
they usually will resolve on their own
most are found on the extremities
they are composed mostly of dilated veins

The correct answer is Choice C


Hemangioma of Infancy (HOI) (also known as strawberry, cherry, capillary hemangioma) is a benign vascular neoplasm and is the most common tumor
of infancy. HOIs are composed of proliferating, plump endothelial cells. Early in proliferation, the cells are in disarray, but, with time, they form vascular
spaces and channels replete with blood cells. These benign-appearing endothelial cells produce limited basement membrane structures. HOIs assume a
lobular architecture as proliferation slows and ends. Consequently, the HOIs are not composed mostly of dilated veins (Choice E).
HOIs are soft, bright red to deep purple and compressible. On diascopy, does not blanch completely and on exam, clinically presents as a nodule or
plaque, 1 to 8 cm in size. Sixty percent of cutaneous hemangiomas occur on the head and neck, 25% on the trunk, and 15% on the extremities. So, the
majority of the HOIs are not found on the extremities (Choice C).Hemangiomas also can occur in extracutaneous sites, including the liver,
gastrointestinal tract, larynx, CNS, pancreas, gall bladder, thymus, spleen, lymph nodes, lung, urinary bladder, and adrenal glands.
The incidence in newborns is between 1 and 2.5%; in white children by 1 year of age it is 10%. Females more affected than males by a 3 to 1 ratio.
HOI is a localized proliferative process of angioblastic mesenchyme. The initial proliferative phase lasts from 3 to 9 months, sometimes more. HOI
usually enlarge rapidly during the first year. In a subsequent phase of involution about the 95% of HOI regress, and this occurs gradually over 2 to 6
years and is usually complete by the age of 10 that practically excludes presence of the HOI in the teenage years (Choice B). Involution varies greatly
between individuals and is not correlated with size, location, or appearance of the lesion.
Surgery (Choice A) is an incorrect answer because it is not a treatment of choice. Surgery is indicated in HOI that obstruct vital structures (eyes, ears,
larynx) and those (<1%) that are life threatening. Other options include topical intralesional steroids, pulsed dye laser and cryosurgery. However,
treatment modalities mentioned above can have serious side effects such as scarring (laser, surgery, cryodestruction) and skin atrophy (intralesional
steroids). The rate of spontanious resolution is greater than 50% and it is usually complete earlier than their 20s.
New research has now shown that Propanolol, if used within the first year of life, can have some benefit effects in the natural course of HOI. Dosing
includes 2 mg kg day given in divided doses every 12 hours. Blood pressure, pulse and blood sugar are mandatory tests that should be done in office
when starting this medication.

A 15 year old male presents to your office complaining of acne that failed to resolve with the topical tretinoin that you prescribed several months ago.
He reports that he used it for 2 weeks, but it did not help, so he stopped using it. On physical examination, you appreciate both open and closed
comedones with areas of pustule formation and scattered scarring. The lesions are mainly distributed across the forehead and cheeks with the
appearance of scattered lesions on the chest and upper back. The skin on his face also appears very oily.
What is the next best step in treatment of his acne?
1) isotretinoin (Accutane)
2) topical tretinoin
3) topical benzoyl peroxide

4) oral tetracycline
5) a combination of oral tetracycline and topical tretinoin

The correct answer is choice B.

This teenager has acne vulgaris, a condition common among adolescents. While supportive treatment and good hygiene may suffice for milder forms of
acne vulgaris, the most severe forms can be physically disfiguring and psychosocially impairing.

Topical retinoids, including tretinoin, should be the first-line agents used. They are very effective against acne vulgaris but need to be used consistently
on a daily basis for at least 6-8 weeks before improvement can be seen. The patient in this vignette ceased use of the topical tretinoin after 2 weeks,
which is not long enough duration to assess whether the treatment was effective. He should be advised to continue daily treatment for 6-8 weeks then
return to your office for assessment of his acne.

Other first line topical agents include benzoyl peroxide (choice C) which has an antimicrobial properties and topical antibiotics such as clindamycin and
erythromycin. Benzoyl peroxide is an effective agent against acne but is not as effective as the topical retinoids. Topical antibiotics should not be used
as monotherapy against acne vulgaris and are most effective in combination with other topical agents.

Oral antibiotics such as tetracycline (choice D) are a possible treatment choice after a patient has failed initial topical therapies. Tetracycline acts to
inhibit growth of Propionibacterium acnes, the main bacterial cause of acne vulgaris. In this vignette, the patient has not yet failed initial therapy, as he
was noncompliant with the use of the topical tretinoin. Combination treatment along with topical retinoids (choice E) is also a good option for treatment
once topical retinoids alone have failed.

Isotretinoin (choice A; brand name Accutane) is a very effective choice of treatment for severe, nodulocystic acne vulgaris that has been refractory to all
other treatment options. Isotretinoin has many side effects including severe irritation of the eyes, mouth, and underlying skin. Pre-existing liver disease
and hypercholesterolemia must be excluded prior to treatment initiation due to the possibility of increased serum cholesterol and triglyceride levels with
the drug. Additionally, pregnancy must be ruled out prior to the start of treatment, and patients must pledge not to become pregnant during the
course, as isoretinoin is highly teratogenic to the developing fetus.

A 2 year old female infant presents to your office for a routine well child check. Her mother asks you what can be done about the birthmark on her
child's arm. She tells you that it has been there since birth and seems to be getting larger instead of going away as she has previously been told it
would by another doctor. You note that there is a 5 cm, raised, sharply demarcated area of erythema on the right lower arm that has increased in size
from her 18 month visit. There is no associated ulceration, bleeding, or discharge. No other lesions or birthmarks are noted, and her physical exam is
otherwise unremarkable.
Which of the following is the most appropriate next step in management of this lesion?
1)
2)
3)
4)
5)

intralesional corticosteroid injection


a course of oral steroids
laser treatment
surgical removal
observation

The correct answer is choice E.

This vignette describes a hemangioma. Hemangiomas are very common in infancy, occurring in up to 2% of newborns. They are benign vascular
hamartomas and usually manifest by 2 months of age. The most common areas affected are the face, back, and chest, but they can occur anywhere
on the skin as well as on internal structures. For most hemangiomas, no treatment is necessary as they will spontaneously involute as the child grows
older. The vast majority involute by 9 years of age, with up to 60% involuting by 5 years of age.

Hemangiomas can become symptomatic depending on their location. In some patients, they may begin to ulcerate and cause pain. If they are located
on the lower face and neck, evaluation should be undertaken to ensure that the hemangioma does not extend inward toward the airway.

Laser treatment (answer choice C; also known as pulsed dye laser, or PDL) can be helpful in the initial stages of the hemangioma to reduce the rate of
growth. It is most commonly used to treat large, rapidly growing hemangiomas that threaten to intrude on other vital structures such as the eyes or
airway. This is rarely indicated, however, for most hemangiomas.

Steroids, both oral and intralesional (answer choices A and B), are effective options for the reduction of size of hemangiomas. However, steroids are
not without risk and should not be used unless the hemangioma is grossly disfiguring or intruding on other vital structures, as above.

Surgical removal (answer choice D), as with the above answers, is an option for hemangiomas that pose a health risk to the patient or are refractory to
other treatments.

A 30-year-old male presents at your clinic complaining of extremely itchy papulovesicular lesions on his extremities, knees, elbows and buttocks for one
year. The blisters vary in size from very small up to 1 cm across. Direct immunofluorescence staining of the lesions show IgA deposition at the dermal
epidermal junction.
The most probable diagnosis is:
1)
2)
3)
4)
5)

Pemphigus vulgaris
Bullous pemphigoid
Dermatitis herpetiformis
Nummular eczema
Intraepidermal neutrophilic IgA dermatosis

The correct answer is Choice C.

Also known as Duhring's Disease, dermatitis herpetiformis (Choice C) is a chronic blistering skin condition, characterized by intensely itchy, chronic
papulovesicular eruptions. The age of onset is usually about 15-40 years old and the condition is not gender-specific.

Dermatitis herpetiformis responds well to medication and changes in diet. Dapsone is an effective treatment for most patients; however, this treatment
has no effect on any intestinal damage that might be present. A strict gluten-free diet must therefore also be followed, and this will usually be a lifelong
requirement. This will reduce any associated intestinal damage, and the risk of other complications. After some time on a gluten-free diet, the dosage of
dapsone can usually be reduced or even stopped, although this can take up to anything from 1 to 3 years.Diagnosis is confirmed by a simple blood test
for IgA antibodies, and by a skin biopsy in which the pattern of IgA deposits in the dermal papillae, revealed by direct immunofluorescence,
distinguishes it from linear IgA bullous dermatosis (Choice B) and other forms of dermatitis.

Pemphigus vulgaris (Choice A) is associated with circulating IgG antibodies.

Nummular eczema (Choice D) is characterized by round or oval-shaped itchy lesions and is most common in people in their 60's.

Intraepidermal neutrophilic IgA dermatosis (Choice E) is characterized histologically by intraepidermal bullae with neutrophils, some eosinophils, and
acantholysis.

A 16 year old boy comes to your clinic for evaluation of a skin rash. The patient first noticed a large, scaly, pink, oval-shaped lesion on his trunk 6 days
ago. He has since developed numerous smaller, oval-shaped, pink, papular lesions on his trunk arranged along the cutaneous skin lines. History is
significant for a low-grade fever and sore throat a few days before onset of the rash for which he was given oral penicillin.
Which of the following is the most likely diagnosis for this patient?
1)
2)
3)
4)
5)

drug eruption
viral exanthem
pityriasis rosea
psoriasis
tinea corporis

The correct answer is choice C.

This is a classic course of pityriasis rosea, a benign dermatological eruption that commonly occurs in childhood and adolescents. The cause of pityriasis
rosea is unknown, but different viruses have been hypothesized to cause the rash with current thought focused on human herpesvirus 7.

The rash of pityriasis rosea generally begins with the "herald patch," a large, ovoid, scaly patch anywhere on the body, then progress into multiple,
symmetric, smaller, pink, ovoid, scaly lesions commonly on the trunk and arms. The distribution of these smaller lesions often follows the cutaneous
skin lines, forming what is known as the "Christmas tree" pattern of pityriasis rosea. A viral-like prodrome of fever, sore throat, and malaise has been
reported to precede the rash but is not required for diagnosis.

Treatment for pityriasis rosea is generally symptomatic. Antihistamines can be given if there is pruritis, and lotions or emollients can be used for the
scaling nature of the rash. Topical corticosteroids are also sometimes used for the pruritis, if present. Antifungals have no role in treatment. There are

no lasting sequelae of pityriasis rosea except for some minor hypo- or hyperpigmentation that may persist for a few weeks after the rash resolves.

A drug eruption (choice A) may be a tempting answer given the recent use of penicillin, but the history and clinical course is classic for pityriasis rosea.
Drug eruptions caused by penicillin are also usually type II hypersensitivity reactions and are characterized by hemolysis and purpura.

Viral exanthems (choice B) can look similar to pityriasis rosea, and the history of fever and sore throat may lead one to think this is all due to a viral
process such as EBV or CMV, but classic viral exanthems are generally more morbilliform in nature and without the scaly appearance and "Christmas
tree" distribution.

Psoriasis (choice D) is a skin disorder characterized by erythematous papules that coalesce into large plaques with a silvery scale appearance. It is
usually found on the scalp, knees, and elbows.

Tinea corporis (choice E) is commonly misdiagnosed when pityriasis rosea is present, as the lesions can be somewhat similar. Tinea corporis, however,
is a fungal infection in which dry, erythematous, scaly plaques with a characteristic central clearing develop. Diagnosis can be made by examining
KOH wet mount preparations of scrapings of the lesions under a microscope

Anda mungkin juga menyukai